Could a 19.25mm revolver actually exist?





.everyoneloves__top-leaderboard:empty,.everyoneloves__mid-leaderboard:empty,.everyoneloves__bot-mid-leaderboard:empty{ margin-bottom:0;
}







25












$begingroup$


I'm working on writing a story wherein there is a revolver inspired by Guillermo del Toro's Hellboy's samaritan. My question is; can such a gun, as described below, work? Or would it be a bone breaker like Hellboy's Samaritan?




Hellboy's Samaritan; 22mm four shot revolver which supposedly would
break a man's arm. Further details largely unknown.



VD Revolver(VDR)(from my story); 19.25mm, 6 shot revolver. Cylinder
between 3 and 3.5 inches wide with a 12" barrel, the last two inches
of that a muzzle break venting straight back. Miscellaneous details;
fires from the bottom of the cylinder for the lowest possible bore
axis, swing out cylinder, single action only, ten pound weight.




A little back story on it, and my story's world. Robotic augmentation is common, this gun first made by a man with robotic arms just because he could. The government then, to commemorate Victory Day (VD), modified it only slightly and issued it to exemplary officer's as a type of recognition of valor. The original was based largely on old world doctrines of stopping revolvers, the government version kept the same doctrine. One shot enough to kill anything no matter how augmented. Of course since only augmented individuals could fire a compromise was later made when a human wanted to fire one. For whatever reason a few made it onto the civilian market and from there tweaked. The original cartridge; 3000 grain bullet weight at 1250 feet per second, was then down loaded to a 1500 grain bullet weight at 800 feet per second. Still monstrous but only actually four thirds the calculated recoil of our largest handgun round. The S&W 500 with a 700 grain hardcast. The VDR similar in size to the Magnum Research BFR. Now of course it is impractical, and maybe absurd, but the character using them ain't in it for reason. The reason they choose these specifically is that they simply want to. They just like them, they get aroused by them. Physical limb replacement is out but the character might have nano-tech augmentations, it'd be a long shot though.



So to recap the actual question is; can such a gun, as described, work? Or would it be a bone breaker like Hellboy's Samaritan? The only real requirements to be considered working; that it fires, hits things reasonably accurately within forty yards, that it not tear itself apart, that it doesn't kill or maim operator.



Commenter points;
The character in question does have smaller handguns, just prefers these if possible.



Massive and unwieldy? Yes but we, in this universe, have the BFR and the VDR weight of ten pounds is loaded. Unloaded its 7.5 pounds which is two pounds heavier than the BFR. I can see this problem being dealt with by augmentation or training.



Chamber being about 4 inches long should keep most fingers safe.










share|improve this question











$endgroup$










  • 1




    $begingroup$
    Comments are not for extended discussion; this conversation has been moved to chat.
    $endgroup$
    – L.Dutch
    May 24 at 2:36






  • 1




    $begingroup$
    Is it a requirement to only use kinetic energy to kill? Otherwise there are real life 40 mm revolvers around...
    $endgroup$
    – vsz
    May 24 at 6:07










  • $begingroup$
    The other practical problem with such big handguns is that the barrel is too short for all the powder to burn up in the barrel. You wind up getting hellacious (and very wasteful!) muzzle flash.
    $endgroup$
    – RonJohn
    May 25 at 1:25










  • $begingroup$
    @vsz it's not a handgun.
    $endgroup$
    – RonJohn
    May 25 at 1:26










  • $begingroup$
    @RonJohn that would actually be the reason for a twelve inch barrel to burn up most of the powder. Maybe not all but a fair portion, at least in theory.
    $endgroup$
    – neo
    May 26 at 14:00


















25












$begingroup$


I'm working on writing a story wherein there is a revolver inspired by Guillermo del Toro's Hellboy's samaritan. My question is; can such a gun, as described below, work? Or would it be a bone breaker like Hellboy's Samaritan?




Hellboy's Samaritan; 22mm four shot revolver which supposedly would
break a man's arm. Further details largely unknown.



VD Revolver(VDR)(from my story); 19.25mm, 6 shot revolver. Cylinder
between 3 and 3.5 inches wide with a 12" barrel, the last two inches
of that a muzzle break venting straight back. Miscellaneous details;
fires from the bottom of the cylinder for the lowest possible bore
axis, swing out cylinder, single action only, ten pound weight.




A little back story on it, and my story's world. Robotic augmentation is common, this gun first made by a man with robotic arms just because he could. The government then, to commemorate Victory Day (VD), modified it only slightly and issued it to exemplary officer's as a type of recognition of valor. The original was based largely on old world doctrines of stopping revolvers, the government version kept the same doctrine. One shot enough to kill anything no matter how augmented. Of course since only augmented individuals could fire a compromise was later made when a human wanted to fire one. For whatever reason a few made it onto the civilian market and from there tweaked. The original cartridge; 3000 grain bullet weight at 1250 feet per second, was then down loaded to a 1500 grain bullet weight at 800 feet per second. Still monstrous but only actually four thirds the calculated recoil of our largest handgun round. The S&W 500 with a 700 grain hardcast. The VDR similar in size to the Magnum Research BFR. Now of course it is impractical, and maybe absurd, but the character using them ain't in it for reason. The reason they choose these specifically is that they simply want to. They just like them, they get aroused by them. Physical limb replacement is out but the character might have nano-tech augmentations, it'd be a long shot though.



So to recap the actual question is; can such a gun, as described, work? Or would it be a bone breaker like Hellboy's Samaritan? The only real requirements to be considered working; that it fires, hits things reasonably accurately within forty yards, that it not tear itself apart, that it doesn't kill or maim operator.



Commenter points;
The character in question does have smaller handguns, just prefers these if possible.



Massive and unwieldy? Yes but we, in this universe, have the BFR and the VDR weight of ten pounds is loaded. Unloaded its 7.5 pounds which is two pounds heavier than the BFR. I can see this problem being dealt with by augmentation or training.



Chamber being about 4 inches long should keep most fingers safe.










share|improve this question











$endgroup$










  • 1




    $begingroup$
    Comments are not for extended discussion; this conversation has been moved to chat.
    $endgroup$
    – L.Dutch
    May 24 at 2:36






  • 1




    $begingroup$
    Is it a requirement to only use kinetic energy to kill? Otherwise there are real life 40 mm revolvers around...
    $endgroup$
    – vsz
    May 24 at 6:07










  • $begingroup$
    The other practical problem with such big handguns is that the barrel is too short for all the powder to burn up in the barrel. You wind up getting hellacious (and very wasteful!) muzzle flash.
    $endgroup$
    – RonJohn
    May 25 at 1:25










  • $begingroup$
    @vsz it's not a handgun.
    $endgroup$
    – RonJohn
    May 25 at 1:26










  • $begingroup$
    @RonJohn that would actually be the reason for a twelve inch barrel to burn up most of the powder. Maybe not all but a fair portion, at least in theory.
    $endgroup$
    – neo
    May 26 at 14:00














25












25








25


6



$begingroup$


I'm working on writing a story wherein there is a revolver inspired by Guillermo del Toro's Hellboy's samaritan. My question is; can such a gun, as described below, work? Or would it be a bone breaker like Hellboy's Samaritan?




Hellboy's Samaritan; 22mm four shot revolver which supposedly would
break a man's arm. Further details largely unknown.



VD Revolver(VDR)(from my story); 19.25mm, 6 shot revolver. Cylinder
between 3 and 3.5 inches wide with a 12" barrel, the last two inches
of that a muzzle break venting straight back. Miscellaneous details;
fires from the bottom of the cylinder for the lowest possible bore
axis, swing out cylinder, single action only, ten pound weight.




A little back story on it, and my story's world. Robotic augmentation is common, this gun first made by a man with robotic arms just because he could. The government then, to commemorate Victory Day (VD), modified it only slightly and issued it to exemplary officer's as a type of recognition of valor. The original was based largely on old world doctrines of stopping revolvers, the government version kept the same doctrine. One shot enough to kill anything no matter how augmented. Of course since only augmented individuals could fire a compromise was later made when a human wanted to fire one. For whatever reason a few made it onto the civilian market and from there tweaked. The original cartridge; 3000 grain bullet weight at 1250 feet per second, was then down loaded to a 1500 grain bullet weight at 800 feet per second. Still monstrous but only actually four thirds the calculated recoil of our largest handgun round. The S&W 500 with a 700 grain hardcast. The VDR similar in size to the Magnum Research BFR. Now of course it is impractical, and maybe absurd, but the character using them ain't in it for reason. The reason they choose these specifically is that they simply want to. They just like them, they get aroused by them. Physical limb replacement is out but the character might have nano-tech augmentations, it'd be a long shot though.



So to recap the actual question is; can such a gun, as described, work? Or would it be a bone breaker like Hellboy's Samaritan? The only real requirements to be considered working; that it fires, hits things reasonably accurately within forty yards, that it not tear itself apart, that it doesn't kill or maim operator.



Commenter points;
The character in question does have smaller handguns, just prefers these if possible.



Massive and unwieldy? Yes but we, in this universe, have the BFR and the VDR weight of ten pounds is loaded. Unloaded its 7.5 pounds which is two pounds heavier than the BFR. I can see this problem being dealt with by augmentation or training.



Chamber being about 4 inches long should keep most fingers safe.










share|improve this question











$endgroup$




I'm working on writing a story wherein there is a revolver inspired by Guillermo del Toro's Hellboy's samaritan. My question is; can such a gun, as described below, work? Or would it be a bone breaker like Hellboy's Samaritan?




Hellboy's Samaritan; 22mm four shot revolver which supposedly would
break a man's arm. Further details largely unknown.



VD Revolver(VDR)(from my story); 19.25mm, 6 shot revolver. Cylinder
between 3 and 3.5 inches wide with a 12" barrel, the last two inches
of that a muzzle break venting straight back. Miscellaneous details;
fires from the bottom of the cylinder for the lowest possible bore
axis, swing out cylinder, single action only, ten pound weight.




A little back story on it, and my story's world. Robotic augmentation is common, this gun first made by a man with robotic arms just because he could. The government then, to commemorate Victory Day (VD), modified it only slightly and issued it to exemplary officer's as a type of recognition of valor. The original was based largely on old world doctrines of stopping revolvers, the government version kept the same doctrine. One shot enough to kill anything no matter how augmented. Of course since only augmented individuals could fire a compromise was later made when a human wanted to fire one. For whatever reason a few made it onto the civilian market and from there tweaked. The original cartridge; 3000 grain bullet weight at 1250 feet per second, was then down loaded to a 1500 grain bullet weight at 800 feet per second. Still monstrous but only actually four thirds the calculated recoil of our largest handgun round. The S&W 500 with a 700 grain hardcast. The VDR similar in size to the Magnum Research BFR. Now of course it is impractical, and maybe absurd, but the character using them ain't in it for reason. The reason they choose these specifically is that they simply want to. They just like them, they get aroused by them. Physical limb replacement is out but the character might have nano-tech augmentations, it'd be a long shot though.



So to recap the actual question is; can such a gun, as described, work? Or would it be a bone breaker like Hellboy's Samaritan? The only real requirements to be considered working; that it fires, hits things reasonably accurately within forty yards, that it not tear itself apart, that it doesn't kill or maim operator.



Commenter points;
The character in question does have smaller handguns, just prefers these if possible.



Massive and unwieldy? Yes but we, in this universe, have the BFR and the VDR weight of ten pounds is loaded. Unloaded its 7.5 pounds which is two pounds heavier than the BFR. I can see this problem being dealt with by augmentation or training.



Chamber being about 4 inches long should keep most fingers safe.







reality-check technology physics weapons engineering






share|improve this question















share|improve this question













share|improve this question




share|improve this question








edited May 27 at 21:59









Martin Schröder

3135 silver badges15 bronze badges




3135 silver badges15 bronze badges










asked May 23 at 14:01









neoneo

1922 silver badges8 bronze badges




1922 silver badges8 bronze badges











  • 1




    $begingroup$
    Comments are not for extended discussion; this conversation has been moved to chat.
    $endgroup$
    – L.Dutch
    May 24 at 2:36






  • 1




    $begingroup$
    Is it a requirement to only use kinetic energy to kill? Otherwise there are real life 40 mm revolvers around...
    $endgroup$
    – vsz
    May 24 at 6:07










  • $begingroup$
    The other practical problem with such big handguns is that the barrel is too short for all the powder to burn up in the barrel. You wind up getting hellacious (and very wasteful!) muzzle flash.
    $endgroup$
    – RonJohn
    May 25 at 1:25










  • $begingroup$
    @vsz it's not a handgun.
    $endgroup$
    – RonJohn
    May 25 at 1:26










  • $begingroup$
    @RonJohn that would actually be the reason for a twelve inch barrel to burn up most of the powder. Maybe not all but a fair portion, at least in theory.
    $endgroup$
    – neo
    May 26 at 14:00














  • 1




    $begingroup$
    Comments are not for extended discussion; this conversation has been moved to chat.
    $endgroup$
    – L.Dutch
    May 24 at 2:36






  • 1




    $begingroup$
    Is it a requirement to only use kinetic energy to kill? Otherwise there are real life 40 mm revolvers around...
    $endgroup$
    – vsz
    May 24 at 6:07










  • $begingroup$
    The other practical problem with such big handguns is that the barrel is too short for all the powder to burn up in the barrel. You wind up getting hellacious (and very wasteful!) muzzle flash.
    $endgroup$
    – RonJohn
    May 25 at 1:25










  • $begingroup$
    @vsz it's not a handgun.
    $endgroup$
    – RonJohn
    May 25 at 1:26










  • $begingroup$
    @RonJohn that would actually be the reason for a twelve inch barrel to burn up most of the powder. Maybe not all but a fair portion, at least in theory.
    $endgroup$
    – neo
    May 26 at 14:00








1




1




$begingroup$
Comments are not for extended discussion; this conversation has been moved to chat.
$endgroup$
– L.Dutch
May 24 at 2:36




$begingroup$
Comments are not for extended discussion; this conversation has been moved to chat.
$endgroup$
– L.Dutch
May 24 at 2:36




1




1




$begingroup$
Is it a requirement to only use kinetic energy to kill? Otherwise there are real life 40 mm revolvers around...
$endgroup$
– vsz
May 24 at 6:07




$begingroup$
Is it a requirement to only use kinetic energy to kill? Otherwise there are real life 40 mm revolvers around...
$endgroup$
– vsz
May 24 at 6:07












$begingroup$
The other practical problem with such big handguns is that the barrel is too short for all the powder to burn up in the barrel. You wind up getting hellacious (and very wasteful!) muzzle flash.
$endgroup$
– RonJohn
May 25 at 1:25




$begingroup$
The other practical problem with such big handguns is that the barrel is too short for all the powder to burn up in the barrel. You wind up getting hellacious (and very wasteful!) muzzle flash.
$endgroup$
– RonJohn
May 25 at 1:25












$begingroup$
@vsz it's not a handgun.
$endgroup$
– RonJohn
May 25 at 1:26




$begingroup$
@vsz it's not a handgun.
$endgroup$
– RonJohn
May 25 at 1:26












$begingroup$
@RonJohn that would actually be the reason for a twelve inch barrel to burn up most of the powder. Maybe not all but a fair portion, at least in theory.
$endgroup$
– neo
May 26 at 14:00




$begingroup$
@RonJohn that would actually be the reason for a twelve inch barrel to burn up most of the powder. Maybe not all but a fair portion, at least in theory.
$endgroup$
– neo
May 26 at 14:00










7 Answers
7






active

oldest

votes


















46














$begingroup$

You might be surprised to learn that even the full power version of this gun could be fired by an experienced shooter without physical augmentation.



It's all about gun design.



First, there's porting. This is a pair or symmetrical set of openings in the top of the barrel, near the muzzle (typically to either side of the front sight). Propellant gas, still under high pressure, exits from these openings just before and as the bullet exits the muzzle, and the resulting rocket thrust greatly reduces muzzle flip, which is a very large part of the equation in terms of controlling recoil in a hard-kicking handgun.



Second, there's mass. A big gun like this will be heavy anyway, but if no effort is made to lighten it (to ease carrying), or mass intentionally added (tungsten inserts in the frame, extra long, extra heavy barrel), every ounce left in the gun is soaking up recoil momentum and lowering the recoil velocity.



Third, there's grip design. Look a the grips on a BFR (designed for rounds as powerful as .45-70 Government). They're classic single action grip shape, a design that puts most of the recoil into raising the muzzle (opposed by the porting and mass of the gun), so the gun goes up when fired, instead of back into the shooter's face (as has been a demonstrated problem with .500 S&W in a double action revolver - sometimes even causing double firing when gun movement operates the trigger a second time).



Combine all three, and you wind up with an oversize BFR style single-action revolver that anyone who knows what to expect and is in good condition (with adult male upper body strength, at least) can fire safely. If they don't develop a horrible flinch the first few shots, there's no reason it couldn't be accurate, either, within its limits.






share|improve this answer











$endgroup$











  • 2




    $begingroup$
    This is actually very helpful. I didn't have any concrete examples though i knew muzzle flip was the enemy. That was why I wrote in that it fires from the bottom of the cylinder and why it weighs so much. As well as why the last two inches of barrel are a muzzle brake.
    $endgroup$
    – neo
    May 23 at 14:54






  • 3




    $begingroup$
    FWIW, this is approximately comparable, in terms of experience, to firing a 12 ga. slug load out of a double barrel pistol. You bullet is a little heavier, similar velocity, but the double pistol is a LOT lighter. People do fire those without damage...
    $endgroup$
    – Zeiss Ikon
    May 23 at 14:56






  • 1




    $begingroup$
    Saw a guy fire a single shot .45-70 Government pistol once. It was top break and he had a strap attached to the barrel that wrapped around his weak side hand. Even with a glove and leaning into it, it was still a punishing round to fire.
    $endgroup$
    – zeta-band
    May 23 at 23:17






  • 2




    $begingroup$
    The real issue with weight comes from the cylinder, you are talking about a huge cylinder to hold six 22mm rounds. 8lbs of cylinder is not unreasonable assumption. You are also talking about 4-6 times as much powder as a 12 gauge slug. Fireable, sure, almost anything is fireable, reliably aimable not a chance.
    $endgroup$
    – John
    May 23 at 23:44








  • 1




    $begingroup$
    Yeah -- this sounds like a stopping revolver, basically. I could see this being a blackpowder era thing, but why would you need a cartridge that heavy and punishing in smokeless, considering that you aren't going to be able to use anywhere near the full case volume?
    $endgroup$
    – Shalvenay
    May 24 at 23:01



















25














$begingroup$

Yes



28mm revolvers exist they are just not practical weapons (~14lbs).
a 22mm would be almost identical to a 8guage shotgun shell, except it will have more powder.



enter image description here



Hellboy's gun has relatively low power you don't want to copy it.
The length of the cartridge also matters as much as the diameter, a wide short cartridge is not going to pack much punch. Hellboy's cartridges are wide but very short, I would not expect more punch than a large pistol or mid range shotgun.



Your homebrewed cartridges are a bit better, almost identical to a 8 gauge shotgun shell (21.21mm). Homemade 12gauge shotgun revolvers exist and weight around 11lbs, you can drop some weight by using better materials. note your real problem is you have a lot of wasted metal making a cylinder big enough for 6 large diameter rounds, the cylinder needs to be wide enough to fit them all at equal distance which makes for a very large diameter cylinder. That is a lot of wasted metal in the center of the cylinder, and that adds a lot of weight. That is why the largest handguns often only hold 5 rounds, it has a huge impact on weight.



enter image description here



But you could make your bullets a lot better.



So how do you really take advantage of augmentation, use a necked round, you can drastically improve the power for similar recoil. What extreme can you get wit that. well the largest common rifle round is the .50bmg, is 12.7 X 99mm and packs significantly more punch than a an 8 gauge shotgun will.



A .50 bmg handgun also exists (see image) .50 bmg is necked rifle round and produces 65,000 psi (450,000 kPa) and 2-3000 foot pounds of force so a measly 1200 is no problem. Even hand made .50bmg revolvers exist but are comically heavy. A fireable single shot .50 bmg production handgun also exists, THUNDER 50 BMG, it weights 12lbs empty although a significant amount of its mass is recoil absorption.



enter image description here



So in the end you are looking at something in the 10-13 lb range. could a normal human fire it without aid, no, not in any way reliable or combat useful. But an augmented human likely could you just need something to absorb recoil and carry the weight.






share|improve this answer











$endgroup$















  • $begingroup$
    Thanks, those are some interesting examples I was unable to turn up myself.
    $endgroup$
    – neo
    May 23 at 15:02










  • $begingroup$
    The Thunder .50 is essentially a hand cannon: it's got all the usual features of an artillery piece (pneumatic recoil absorption, muzzle brake, etc.) in a nominally hand-held form factor.
    $endgroup$
    – Mark
    May 23 at 20:57






  • 1




    $begingroup$
    @Mark, it is also hilarious to watch someone fire it, you can almost see their body screaming NOPE afterwards. And this has much less recoil than a 22mm would have.
    $endgroup$
    – John
    May 23 at 23:49






  • 1




    $begingroup$
    There have historically been problems using necked cartridges in revolver. Case expansion (specifically in the shoulder) sets the cartridge back, and then stays that way, locking up the cylinder. Yes, necked case revolvers have been made to work, but it require scrupulous cleaning and oil removal inside the chambers. If you need to reload two or three times in combat, you're virtually certain to run into this issue.
    $endgroup$
    – Zeiss Ikon
    May 24 at 11:23










  • $begingroup$
    I suspect Hellboy's ammo is just fine, and it has the large caliber because it isn't just a slug. It's like how modern fighter jets really struggle to have ammo smaller than 25mm... the bullets are tiny artillery shells with their own HE, shrapnel and fuzes (yes Z). In fact fighter-jet "training blanks" actually are plain slugs, so still pretty deadly. Hellboy's gun only needs to get the bullet there, the bullet itself does all the heavy lifting...
    $endgroup$
    – Harper
    May 26 at 15:24



















8














$begingroup$

You Betcha



If we temporarily ignore the capacity of the wielder to carry the monster, you could weld a hand grip and stock to a Howitzer. Maybe only the Hulk could use it. But it would work just dandy.



So, having proven the premise is possible, let's focus on plausible. I give you:



Recoilless rifles




A recoilless rifle, recoilless launcher or recoilless gun, sometimes abbreviated "RR" or "RCL" (for ReCoilLess) is a type of lightweight artillery system or man-portable launcher that is designed to eject some form of countermass such as propellant gas from the rear of the weapon at the moment of firing, creating forward thrust that counteracts most of the weapon's recoil.




Now, I know what you're thinking. "These are usually shoulder-mounted critters like anti-tank rockets because, well, the back-blast might cause mild skin irritation on the user's face." And you'd be completely correct. But I'm thinking that all we need is some kind of shield, right? Let's think Logan's Run.



Copyright MGM



copyright MGM



Having done it's work, the back-blast is directed harmlessly out of the user's way. Just because it has the potential to start forest fires shouldn't detract from the natural beauty and form of the weapon. Besides, who doesn't like a picture of Jenny Agutter?



I'm going to leave the ability of the user to hold the weight of the weapon up to the OP. Frankly, I'd hate to hold it on some Evil Genius who's decided to start monologing. No matter how strong you are, that much weight would become uncomfortable mighty quickly. Isometrics, anyone?



Edit:  Daniel B brings up a good point. There's an out-of-manufacture class of weapons that could solve most if not all of your problems: the Gyrojet.




The Gyrojet is a family of unique firearms developed in the 1960s named for the method of gyroscopically stabilizing its projectiles. Rather than inert bullets, Gyrojets fire small rockets called Microjets which have little recoil and do not require a heavy barrel or chamber to resist the pressure of the combustion gases. Velocity on leaving the tube was very low, but increased to around 1,250 feet per second (380 m/s) at 30 feet (9.1 m). The result is a very lightweight weapon.




The Gyrojet is the realization of considerable sci-fi fodder, most notably from the Buck Rogers series of comics (1933) and books where the "rocket pistol" was a staple. I'm especially fond of my copy of Philip Francis Nowlan's Amargeddon 2419 A.D.: The Seminal "Buck Rogers" Novel,1 (1978) which has a parallel universe, disintergrator rays, and repulsar beams — and doesn't waste one darn minute trying to explain any of them. Oh, yeah....





1Rendered nearly worthless to anyone but me due to the number of times I've read the book. Yes, I understand there's value in mint condition books — but the true value of a book is discovered by how worn the cover and pages are. Who cares about a book that's never read?






share|improve this answer











$endgroup$











  • 2




    $begingroup$
    who doesn't like a picture of Jenny Agutter Tempting to vote you up just for that remark, but I'm pretty sure it's against the spirit of the rules. :-)
    $endgroup$
    – StephenG
    May 23 at 19:35






  • 1




    $begingroup$
    @JBH: I was going to put an answer involving gyrojets but this answer covers the basic concept without actually naming them. Could you edit in a reference to them?
    $endgroup$
    – Daniel B
    May 24 at 0:20






  • 2




    $begingroup$
    I wish I could upvote twice for the "Just because it has the potential to start forest fires should detract from the natural beauty and form of the weapon." part.
    $endgroup$
    – Renan
    May 24 at 1:44










  • $begingroup$
    @DanielB Done! It actually reminded me of a favorite novel, which I had the chance to mention. Thanks!
    $endgroup$
    – JBH
    May 24 at 4:15










  • $begingroup$
    "Having done it's work, the back-blast is directed harmlessly out of the user's way" and onto the people standing next to him. Or is that what you meant by "Just because it has the potential to start forest fires shouldn't detract from the natural beauty and form of the weapon"?
    $endgroup$
    – RonJohn
    May 27 at 5:22



















8














$begingroup$

I've crunched the numbers a bit, and I have to admit that both variants of the weapon are possible, although very unwieldy. What I don't like here is a narrative part - I just can't imagine someone developing such a revolver as a battlefield weapon.



So I've thought about some corrections. Say, we have an increase in human augmentation, robotics, etc. So there are targets that is very hard to take down. I can imagine a government agency developing a ".758 BMG" necked up wildcat of .50 BMG to deal with heavily armored augmented threats in urban environment. The main weapon it used in is a Barret-type rifle, bullpup, suppressed. The round is subsonic and, perhaps, frangible. It has a muzzle energy of about 14000 joules and delivers most of them to the target without overpenetration. It can be fired by non-augmented human operator as an antimateriel rifle, and by a non-augmented operator standing up. .758 BMG rifle is absolutely valid and can be a normal item in your world.



.758 revolver is an experimental crazy offshoot of this technology. It was envisioned as a sidearm for augmented operators, firing the same round. I'm not sure this weapon was ever accepted and mass-produced. It's just too crazy and unwieldy, with too short a barrel for the round it fires. But there can be a number of prototype examples.



For unaugmented human to use it, it needs severely underpowered rounds. And, to be honest, it sucks. Something like .50 Beowulf carbine would give you about the same power (<3000 joules) in much more comfortable and ergonomic package. And all those carbines are of themselves a compromise in trying to fit a big caliber bullet in short action. So it is very much a novelty gimmick and should be treated as such.



Another part that I don't like is that too underload this cartridge you need to leave A LOT of space in it empty. I'm not sure it is safe. My proposed solution was to make the original .758 cartridge caseless. Without brass it would be shorter - maybe around 3.5 inches as opposed to .50 BMG 5 inches. Then the character would be able to manufacture his own weaker cartridges with lighter bullets from cut down .50 BMG brass.



UPD: I kinda forgot your full-power round was supersonic, but IMO, when you go as heavy as 3000 grains, you can just make it subsonic. I envisioned ".758 Barret rifle" as a high-powered analogue to Russian 9x39 rifles, with heavy rounds, subsonic and suppressed, for urban usage.






share|improve this answer











$endgroup$















  • $begingroup$
    Yes! That's exactly it. A rifle does exist and some nut made a handgun with it's cartridge. The government made a limited run of 100 or in my story. To give as decoration to officers. And you gave me an idea. The original prototype could fire the human safe round and the officer's the more powerful full bullet. Since to be an officer you just about have to be an aug. This gives me two versions, which I wanted to avoid, but it does make more sense. I have to admit you are the only one who has made me seriously consider just not giving it to my Character if they don't have augments.
    $endgroup$
    – neo
    May 24 at 15:18



















2














$begingroup$

I can think of 3 ways to solve this problem:



The first approach is to treat it more like a low calabre RPG and less like an actual bullet. Instead of an instantaneous combustion from a cartridge, making the rounds rocket propelled means that the shot accelerates over time significantly reducing the recoil at the moment of launch. 10lb anti-tank rounds can be fired such as to obtain nearly your desired speed within about 30 feet without creating bone breaking recoil. In contrast, a 19.25mm slug will be about 0.1-0.2lb so making a manageable projectile that can reach 800 ft/sec within inches of leaving your barrel should be pretty doable.



A second approach is to treat it like heavy artillery by using hydraulic recoil compensation. By mounting the barrel and firing assembly to a hydraulic track, you can distribute the recoil over time as the barrel and firing assembly moves backwards on the track.



A third approach is to make the slug an explosive round. Explosives are much less dense than lead; so, a 19.25mm explosive round would weight the same as a much smaller solid round; so, you would need a much smaller cartridge to get your shot up to speed.






share|improve this answer











$endgroup$















  • $begingroup$
    The 1500-3000 grain weight was actually the bullet weight not powder, Very helpful though on the HE aspect, I hadn't thought of that for this particular gun.
    $endgroup$
    – neo
    May 23 at 14:56










  • $begingroup$
    Both bullets and propellant are measured in grains. Since you was talking about downloading, I assumed he meant propellent, but in retrospect, those number are much too high to make since since for that.
    $endgroup$
    – Nosajimiki
    May 23 at 15:07












  • $begingroup$
    Your right, I should have been clearer even though they are too high for charge weights. Edited.
    $endgroup$
    – neo
    May 23 at 15:08



















2














$begingroup$

Absolutely. The trick is, the bullet is a bomb, not a slug. The bullet is a miniature artillery shell with its own destructive mechanism - HEAT, bursting charge+shrapnel, etc.



In fact, that's what requires the large bore - shrinking an "active, fuzed*" artillery shell down to 19mm is quite a challenge**, and this is why modern fighter jets shoot 25 or 30mm rounds.



For Hellboy, this kind of destructive power makes sense given who he's fighting. It also makes it quite an achievement to make his own ammo - the miniaturization.



So the bullet itself is doing most of the heavy lifting - it would be deadly if you handed it to the enemy (presuming this delivery method allowed the fuze* to operate normally).



What does that do for kickback? The gun only needs to get the bullet to the destination. The destruction doesn't depend on imparting a whole bunch of kinetic energy like a normal bullet. That means the launching charge can be smaller, meaning lower kickback. However the large caliber shell necessitates a rather large gun with a lot of mass, which would soak up kickback, making it not bad at all. Anyone who could lift the gun could shoot it.



When Hellboy says "it'll break your arm" he might be exaggerating, or might mean that he's also developed plain old kinetic-energy bullets with a lot of propellant. A bullet that wide would make no sense; surely it would be a sabot.





* Do not edit. The mechanism which tells a bomb or shell when to detonate is called a fuZe. Look it up. A fuSe is something else.



** Part of what makes fuzes a challenge is making them not detonate accidentally. For instance the 16" rounds from Iowa arm from a) 1000++ gees from firing, b) -8 gees from air resistance, and spin from the rifling. Only then, impact+time delay sets it off.






share|improve this answer











$endgroup$























    1














    $begingroup$

    In fact, such guns were actually made, for instance Colt's 10-gauge revolving shotgun: https://www.forgottenweapons.com/colt-1855-10-gauge-revolving-shotgun-at-ria/



    (You may be confusing"revolver", which is a mechanism, with "pistol".)






    share|improve this answer









    $endgroup$


















      Your Answer








      StackExchange.ready(function() {
      var channelOptions = {
      tags: "".split(" "),
      id: "579"
      };
      initTagRenderer("".split(" "), "".split(" "), channelOptions);

      StackExchange.using("externalEditor", function() {
      // Have to fire editor after snippets, if snippets enabled
      if (StackExchange.settings.snippets.snippetsEnabled) {
      StackExchange.using("snippets", function() {
      createEditor();
      });
      }
      else {
      createEditor();
      }
      });

      function createEditor() {
      StackExchange.prepareEditor({
      heartbeatType: 'answer',
      autoActivateHeartbeat: false,
      convertImagesToLinks: false,
      noModals: true,
      showLowRepImageUploadWarning: true,
      reputationToPostImages: null,
      bindNavPrevention: true,
      postfix: "",
      imageUploader: {
      brandingHtml: "Powered by u003ca class="icon-imgur-white" href="https://imgur.com/"u003eu003c/au003e",
      contentPolicyHtml: "User contributions licensed under u003ca href="https://creativecommons.org/licenses/by-sa/4.0/"u003ecc by-sa 4.0 with attribution requiredu003c/au003e u003ca href="https://stackoverflow.com/legal/content-policy"u003e(content policy)u003c/au003e",
      allowUrls: true
      },
      noCode: true, onDemand: true,
      discardSelector: ".discard-answer"
      ,immediatelyShowMarkdownHelp:true
      });


      }
      });















      draft saved

      draft discarded
















      StackExchange.ready(
      function () {
      StackExchange.openid.initPostLogin('.new-post-login', 'https%3a%2f%2fworldbuilding.stackexchange.com%2fquestions%2f147616%2fcould-a-19-25mm-revolver-actually-exist%23new-answer', 'question_page');
      }
      );

      Post as a guest















      Required, but never shown

























      7 Answers
      7






      active

      oldest

      votes








      7 Answers
      7






      active

      oldest

      votes









      active

      oldest

      votes






      active

      oldest

      votes









      46














      $begingroup$

      You might be surprised to learn that even the full power version of this gun could be fired by an experienced shooter without physical augmentation.



      It's all about gun design.



      First, there's porting. This is a pair or symmetrical set of openings in the top of the barrel, near the muzzle (typically to either side of the front sight). Propellant gas, still under high pressure, exits from these openings just before and as the bullet exits the muzzle, and the resulting rocket thrust greatly reduces muzzle flip, which is a very large part of the equation in terms of controlling recoil in a hard-kicking handgun.



      Second, there's mass. A big gun like this will be heavy anyway, but if no effort is made to lighten it (to ease carrying), or mass intentionally added (tungsten inserts in the frame, extra long, extra heavy barrel), every ounce left in the gun is soaking up recoil momentum and lowering the recoil velocity.



      Third, there's grip design. Look a the grips on a BFR (designed for rounds as powerful as .45-70 Government). They're classic single action grip shape, a design that puts most of the recoil into raising the muzzle (opposed by the porting and mass of the gun), so the gun goes up when fired, instead of back into the shooter's face (as has been a demonstrated problem with .500 S&W in a double action revolver - sometimes even causing double firing when gun movement operates the trigger a second time).



      Combine all three, and you wind up with an oversize BFR style single-action revolver that anyone who knows what to expect and is in good condition (with adult male upper body strength, at least) can fire safely. If they don't develop a horrible flinch the first few shots, there's no reason it couldn't be accurate, either, within its limits.






      share|improve this answer











      $endgroup$











      • 2




        $begingroup$
        This is actually very helpful. I didn't have any concrete examples though i knew muzzle flip was the enemy. That was why I wrote in that it fires from the bottom of the cylinder and why it weighs so much. As well as why the last two inches of barrel are a muzzle brake.
        $endgroup$
        – neo
        May 23 at 14:54






      • 3




        $begingroup$
        FWIW, this is approximately comparable, in terms of experience, to firing a 12 ga. slug load out of a double barrel pistol. You bullet is a little heavier, similar velocity, but the double pistol is a LOT lighter. People do fire those without damage...
        $endgroup$
        – Zeiss Ikon
        May 23 at 14:56






      • 1




        $begingroup$
        Saw a guy fire a single shot .45-70 Government pistol once. It was top break and he had a strap attached to the barrel that wrapped around his weak side hand. Even with a glove and leaning into it, it was still a punishing round to fire.
        $endgroup$
        – zeta-band
        May 23 at 23:17






      • 2




        $begingroup$
        The real issue with weight comes from the cylinder, you are talking about a huge cylinder to hold six 22mm rounds. 8lbs of cylinder is not unreasonable assumption. You are also talking about 4-6 times as much powder as a 12 gauge slug. Fireable, sure, almost anything is fireable, reliably aimable not a chance.
        $endgroup$
        – John
        May 23 at 23:44








      • 1




        $begingroup$
        Yeah -- this sounds like a stopping revolver, basically. I could see this being a blackpowder era thing, but why would you need a cartridge that heavy and punishing in smokeless, considering that you aren't going to be able to use anywhere near the full case volume?
        $endgroup$
        – Shalvenay
        May 24 at 23:01
















      46














      $begingroup$

      You might be surprised to learn that even the full power version of this gun could be fired by an experienced shooter without physical augmentation.



      It's all about gun design.



      First, there's porting. This is a pair or symmetrical set of openings in the top of the barrel, near the muzzle (typically to either side of the front sight). Propellant gas, still under high pressure, exits from these openings just before and as the bullet exits the muzzle, and the resulting rocket thrust greatly reduces muzzle flip, which is a very large part of the equation in terms of controlling recoil in a hard-kicking handgun.



      Second, there's mass. A big gun like this will be heavy anyway, but if no effort is made to lighten it (to ease carrying), or mass intentionally added (tungsten inserts in the frame, extra long, extra heavy barrel), every ounce left in the gun is soaking up recoil momentum and lowering the recoil velocity.



      Third, there's grip design. Look a the grips on a BFR (designed for rounds as powerful as .45-70 Government). They're classic single action grip shape, a design that puts most of the recoil into raising the muzzle (opposed by the porting and mass of the gun), so the gun goes up when fired, instead of back into the shooter's face (as has been a demonstrated problem with .500 S&W in a double action revolver - sometimes even causing double firing when gun movement operates the trigger a second time).



      Combine all three, and you wind up with an oversize BFR style single-action revolver that anyone who knows what to expect and is in good condition (with adult male upper body strength, at least) can fire safely. If they don't develop a horrible flinch the first few shots, there's no reason it couldn't be accurate, either, within its limits.






      share|improve this answer











      $endgroup$











      • 2




        $begingroup$
        This is actually very helpful. I didn't have any concrete examples though i knew muzzle flip was the enemy. That was why I wrote in that it fires from the bottom of the cylinder and why it weighs so much. As well as why the last two inches of barrel are a muzzle brake.
        $endgroup$
        – neo
        May 23 at 14:54






      • 3




        $begingroup$
        FWIW, this is approximately comparable, in terms of experience, to firing a 12 ga. slug load out of a double barrel pistol. You bullet is a little heavier, similar velocity, but the double pistol is a LOT lighter. People do fire those without damage...
        $endgroup$
        – Zeiss Ikon
        May 23 at 14:56






      • 1




        $begingroup$
        Saw a guy fire a single shot .45-70 Government pistol once. It was top break and he had a strap attached to the barrel that wrapped around his weak side hand. Even with a glove and leaning into it, it was still a punishing round to fire.
        $endgroup$
        – zeta-band
        May 23 at 23:17






      • 2




        $begingroup$
        The real issue with weight comes from the cylinder, you are talking about a huge cylinder to hold six 22mm rounds. 8lbs of cylinder is not unreasonable assumption. You are also talking about 4-6 times as much powder as a 12 gauge slug. Fireable, sure, almost anything is fireable, reliably aimable not a chance.
        $endgroup$
        – John
        May 23 at 23:44








      • 1




        $begingroup$
        Yeah -- this sounds like a stopping revolver, basically. I could see this being a blackpowder era thing, but why would you need a cartridge that heavy and punishing in smokeless, considering that you aren't going to be able to use anywhere near the full case volume?
        $endgroup$
        – Shalvenay
        May 24 at 23:01














      46














      46










      46







      $begingroup$

      You might be surprised to learn that even the full power version of this gun could be fired by an experienced shooter without physical augmentation.



      It's all about gun design.



      First, there's porting. This is a pair or symmetrical set of openings in the top of the barrel, near the muzzle (typically to either side of the front sight). Propellant gas, still under high pressure, exits from these openings just before and as the bullet exits the muzzle, and the resulting rocket thrust greatly reduces muzzle flip, which is a very large part of the equation in terms of controlling recoil in a hard-kicking handgun.



      Second, there's mass. A big gun like this will be heavy anyway, but if no effort is made to lighten it (to ease carrying), or mass intentionally added (tungsten inserts in the frame, extra long, extra heavy barrel), every ounce left in the gun is soaking up recoil momentum and lowering the recoil velocity.



      Third, there's grip design. Look a the grips on a BFR (designed for rounds as powerful as .45-70 Government). They're classic single action grip shape, a design that puts most of the recoil into raising the muzzle (opposed by the porting and mass of the gun), so the gun goes up when fired, instead of back into the shooter's face (as has been a demonstrated problem with .500 S&W in a double action revolver - sometimes even causing double firing when gun movement operates the trigger a second time).



      Combine all three, and you wind up with an oversize BFR style single-action revolver that anyone who knows what to expect and is in good condition (with adult male upper body strength, at least) can fire safely. If they don't develop a horrible flinch the first few shots, there's no reason it couldn't be accurate, either, within its limits.






      share|improve this answer











      $endgroup$



      You might be surprised to learn that even the full power version of this gun could be fired by an experienced shooter without physical augmentation.



      It's all about gun design.



      First, there's porting. This is a pair or symmetrical set of openings in the top of the barrel, near the muzzle (typically to either side of the front sight). Propellant gas, still under high pressure, exits from these openings just before and as the bullet exits the muzzle, and the resulting rocket thrust greatly reduces muzzle flip, which is a very large part of the equation in terms of controlling recoil in a hard-kicking handgun.



      Second, there's mass. A big gun like this will be heavy anyway, but if no effort is made to lighten it (to ease carrying), or mass intentionally added (tungsten inserts in the frame, extra long, extra heavy barrel), every ounce left in the gun is soaking up recoil momentum and lowering the recoil velocity.



      Third, there's grip design. Look a the grips on a BFR (designed for rounds as powerful as .45-70 Government). They're classic single action grip shape, a design that puts most of the recoil into raising the muzzle (opposed by the porting and mass of the gun), so the gun goes up when fired, instead of back into the shooter's face (as has been a demonstrated problem with .500 S&W in a double action revolver - sometimes even causing double firing when gun movement operates the trigger a second time).



      Combine all three, and you wind up with an oversize BFR style single-action revolver that anyone who knows what to expect and is in good condition (with adult male upper body strength, at least) can fire safely. If they don't develop a horrible flinch the first few shots, there's no reason it couldn't be accurate, either, within its limits.







      share|improve this answer














      share|improve this answer



      share|improve this answer








      edited May 24 at 1:41









      Renan

      68.1k20 gold badges155 silver badges330 bronze badges




      68.1k20 gold badges155 silver badges330 bronze badges










      answered May 23 at 14:50









      Zeiss IkonZeiss Ikon

      9,41616 silver badges42 bronze badges




      9,41616 silver badges42 bronze badges











      • 2




        $begingroup$
        This is actually very helpful. I didn't have any concrete examples though i knew muzzle flip was the enemy. That was why I wrote in that it fires from the bottom of the cylinder and why it weighs so much. As well as why the last two inches of barrel are a muzzle brake.
        $endgroup$
        – neo
        May 23 at 14:54






      • 3




        $begingroup$
        FWIW, this is approximately comparable, in terms of experience, to firing a 12 ga. slug load out of a double barrel pistol. You bullet is a little heavier, similar velocity, but the double pistol is a LOT lighter. People do fire those without damage...
        $endgroup$
        – Zeiss Ikon
        May 23 at 14:56






      • 1




        $begingroup$
        Saw a guy fire a single shot .45-70 Government pistol once. It was top break and he had a strap attached to the barrel that wrapped around his weak side hand. Even with a glove and leaning into it, it was still a punishing round to fire.
        $endgroup$
        – zeta-band
        May 23 at 23:17






      • 2




        $begingroup$
        The real issue with weight comes from the cylinder, you are talking about a huge cylinder to hold six 22mm rounds. 8lbs of cylinder is not unreasonable assumption. You are also talking about 4-6 times as much powder as a 12 gauge slug. Fireable, sure, almost anything is fireable, reliably aimable not a chance.
        $endgroup$
        – John
        May 23 at 23:44








      • 1




        $begingroup$
        Yeah -- this sounds like a stopping revolver, basically. I could see this being a blackpowder era thing, but why would you need a cartridge that heavy and punishing in smokeless, considering that you aren't going to be able to use anywhere near the full case volume?
        $endgroup$
        – Shalvenay
        May 24 at 23:01














      • 2




        $begingroup$
        This is actually very helpful. I didn't have any concrete examples though i knew muzzle flip was the enemy. That was why I wrote in that it fires from the bottom of the cylinder and why it weighs so much. As well as why the last two inches of barrel are a muzzle brake.
        $endgroup$
        – neo
        May 23 at 14:54






      • 3




        $begingroup$
        FWIW, this is approximately comparable, in terms of experience, to firing a 12 ga. slug load out of a double barrel pistol. You bullet is a little heavier, similar velocity, but the double pistol is a LOT lighter. People do fire those without damage...
        $endgroup$
        – Zeiss Ikon
        May 23 at 14:56






      • 1




        $begingroup$
        Saw a guy fire a single shot .45-70 Government pistol once. It was top break and he had a strap attached to the barrel that wrapped around his weak side hand. Even with a glove and leaning into it, it was still a punishing round to fire.
        $endgroup$
        – zeta-band
        May 23 at 23:17






      • 2




        $begingroup$
        The real issue with weight comes from the cylinder, you are talking about a huge cylinder to hold six 22mm rounds. 8lbs of cylinder is not unreasonable assumption. You are also talking about 4-6 times as much powder as a 12 gauge slug. Fireable, sure, almost anything is fireable, reliably aimable not a chance.
        $endgroup$
        – John
        May 23 at 23:44








      • 1




        $begingroup$
        Yeah -- this sounds like a stopping revolver, basically. I could see this being a blackpowder era thing, but why would you need a cartridge that heavy and punishing in smokeless, considering that you aren't going to be able to use anywhere near the full case volume?
        $endgroup$
        – Shalvenay
        May 24 at 23:01








      2




      2




      $begingroup$
      This is actually very helpful. I didn't have any concrete examples though i knew muzzle flip was the enemy. That was why I wrote in that it fires from the bottom of the cylinder and why it weighs so much. As well as why the last two inches of barrel are a muzzle brake.
      $endgroup$
      – neo
      May 23 at 14:54




      $begingroup$
      This is actually very helpful. I didn't have any concrete examples though i knew muzzle flip was the enemy. That was why I wrote in that it fires from the bottom of the cylinder and why it weighs so much. As well as why the last two inches of barrel are a muzzle brake.
      $endgroup$
      – neo
      May 23 at 14:54




      3




      3




      $begingroup$
      FWIW, this is approximately comparable, in terms of experience, to firing a 12 ga. slug load out of a double barrel pistol. You bullet is a little heavier, similar velocity, but the double pistol is a LOT lighter. People do fire those without damage...
      $endgroup$
      – Zeiss Ikon
      May 23 at 14:56




      $begingroup$
      FWIW, this is approximately comparable, in terms of experience, to firing a 12 ga. slug load out of a double barrel pistol. You bullet is a little heavier, similar velocity, but the double pistol is a LOT lighter. People do fire those without damage...
      $endgroup$
      – Zeiss Ikon
      May 23 at 14:56




      1




      1




      $begingroup$
      Saw a guy fire a single shot .45-70 Government pistol once. It was top break and he had a strap attached to the barrel that wrapped around his weak side hand. Even with a glove and leaning into it, it was still a punishing round to fire.
      $endgroup$
      – zeta-band
      May 23 at 23:17




      $begingroup$
      Saw a guy fire a single shot .45-70 Government pistol once. It was top break and he had a strap attached to the barrel that wrapped around his weak side hand. Even with a glove and leaning into it, it was still a punishing round to fire.
      $endgroup$
      – zeta-band
      May 23 at 23:17




      2




      2




      $begingroup$
      The real issue with weight comes from the cylinder, you are talking about a huge cylinder to hold six 22mm rounds. 8lbs of cylinder is not unreasonable assumption. You are also talking about 4-6 times as much powder as a 12 gauge slug. Fireable, sure, almost anything is fireable, reliably aimable not a chance.
      $endgroup$
      – John
      May 23 at 23:44






      $begingroup$
      The real issue with weight comes from the cylinder, you are talking about a huge cylinder to hold six 22mm rounds. 8lbs of cylinder is not unreasonable assumption. You are also talking about 4-6 times as much powder as a 12 gauge slug. Fireable, sure, almost anything is fireable, reliably aimable not a chance.
      $endgroup$
      – John
      May 23 at 23:44






      1




      1




      $begingroup$
      Yeah -- this sounds like a stopping revolver, basically. I could see this being a blackpowder era thing, but why would you need a cartridge that heavy and punishing in smokeless, considering that you aren't going to be able to use anywhere near the full case volume?
      $endgroup$
      – Shalvenay
      May 24 at 23:01




      $begingroup$
      Yeah -- this sounds like a stopping revolver, basically. I could see this being a blackpowder era thing, but why would you need a cartridge that heavy and punishing in smokeless, considering that you aren't going to be able to use anywhere near the full case volume?
      $endgroup$
      – Shalvenay
      May 24 at 23:01













      25














      $begingroup$

      Yes



      28mm revolvers exist they are just not practical weapons (~14lbs).
      a 22mm would be almost identical to a 8guage shotgun shell, except it will have more powder.



      enter image description here



      Hellboy's gun has relatively low power you don't want to copy it.
      The length of the cartridge also matters as much as the diameter, a wide short cartridge is not going to pack much punch. Hellboy's cartridges are wide but very short, I would not expect more punch than a large pistol or mid range shotgun.



      Your homebrewed cartridges are a bit better, almost identical to a 8 gauge shotgun shell (21.21mm). Homemade 12gauge shotgun revolvers exist and weight around 11lbs, you can drop some weight by using better materials. note your real problem is you have a lot of wasted metal making a cylinder big enough for 6 large diameter rounds, the cylinder needs to be wide enough to fit them all at equal distance which makes for a very large diameter cylinder. That is a lot of wasted metal in the center of the cylinder, and that adds a lot of weight. That is why the largest handguns often only hold 5 rounds, it has a huge impact on weight.



      enter image description here



      But you could make your bullets a lot better.



      So how do you really take advantage of augmentation, use a necked round, you can drastically improve the power for similar recoil. What extreme can you get wit that. well the largest common rifle round is the .50bmg, is 12.7 X 99mm and packs significantly more punch than a an 8 gauge shotgun will.



      A .50 bmg handgun also exists (see image) .50 bmg is necked rifle round and produces 65,000 psi (450,000 kPa) and 2-3000 foot pounds of force so a measly 1200 is no problem. Even hand made .50bmg revolvers exist but are comically heavy. A fireable single shot .50 bmg production handgun also exists, THUNDER 50 BMG, it weights 12lbs empty although a significant amount of its mass is recoil absorption.



      enter image description here



      So in the end you are looking at something in the 10-13 lb range. could a normal human fire it without aid, no, not in any way reliable or combat useful. But an augmented human likely could you just need something to absorb recoil and carry the weight.






      share|improve this answer











      $endgroup$















      • $begingroup$
        Thanks, those are some interesting examples I was unable to turn up myself.
        $endgroup$
        – neo
        May 23 at 15:02










      • $begingroup$
        The Thunder .50 is essentially a hand cannon: it's got all the usual features of an artillery piece (pneumatic recoil absorption, muzzle brake, etc.) in a nominally hand-held form factor.
        $endgroup$
        – Mark
        May 23 at 20:57






      • 1




        $begingroup$
        @Mark, it is also hilarious to watch someone fire it, you can almost see their body screaming NOPE afterwards. And this has much less recoil than a 22mm would have.
        $endgroup$
        – John
        May 23 at 23:49






      • 1




        $begingroup$
        There have historically been problems using necked cartridges in revolver. Case expansion (specifically in the shoulder) sets the cartridge back, and then stays that way, locking up the cylinder. Yes, necked case revolvers have been made to work, but it require scrupulous cleaning and oil removal inside the chambers. If you need to reload two or three times in combat, you're virtually certain to run into this issue.
        $endgroup$
        – Zeiss Ikon
        May 24 at 11:23










      • $begingroup$
        I suspect Hellboy's ammo is just fine, and it has the large caliber because it isn't just a slug. It's like how modern fighter jets really struggle to have ammo smaller than 25mm... the bullets are tiny artillery shells with their own HE, shrapnel and fuzes (yes Z). In fact fighter-jet "training blanks" actually are plain slugs, so still pretty deadly. Hellboy's gun only needs to get the bullet there, the bullet itself does all the heavy lifting...
        $endgroup$
        – Harper
        May 26 at 15:24
















      25














      $begingroup$

      Yes



      28mm revolvers exist they are just not practical weapons (~14lbs).
      a 22mm would be almost identical to a 8guage shotgun shell, except it will have more powder.



      enter image description here



      Hellboy's gun has relatively low power you don't want to copy it.
      The length of the cartridge also matters as much as the diameter, a wide short cartridge is not going to pack much punch. Hellboy's cartridges are wide but very short, I would not expect more punch than a large pistol or mid range shotgun.



      Your homebrewed cartridges are a bit better, almost identical to a 8 gauge shotgun shell (21.21mm). Homemade 12gauge shotgun revolvers exist and weight around 11lbs, you can drop some weight by using better materials. note your real problem is you have a lot of wasted metal making a cylinder big enough for 6 large diameter rounds, the cylinder needs to be wide enough to fit them all at equal distance which makes for a very large diameter cylinder. That is a lot of wasted metal in the center of the cylinder, and that adds a lot of weight. That is why the largest handguns often only hold 5 rounds, it has a huge impact on weight.



      enter image description here



      But you could make your bullets a lot better.



      So how do you really take advantage of augmentation, use a necked round, you can drastically improve the power for similar recoil. What extreme can you get wit that. well the largest common rifle round is the .50bmg, is 12.7 X 99mm and packs significantly more punch than a an 8 gauge shotgun will.



      A .50 bmg handgun also exists (see image) .50 bmg is necked rifle round and produces 65,000 psi (450,000 kPa) and 2-3000 foot pounds of force so a measly 1200 is no problem. Even hand made .50bmg revolvers exist but are comically heavy. A fireable single shot .50 bmg production handgun also exists, THUNDER 50 BMG, it weights 12lbs empty although a significant amount of its mass is recoil absorption.



      enter image description here



      So in the end you are looking at something in the 10-13 lb range. could a normal human fire it without aid, no, not in any way reliable or combat useful. But an augmented human likely could you just need something to absorb recoil and carry the weight.






      share|improve this answer











      $endgroup$















      • $begingroup$
        Thanks, those are some interesting examples I was unable to turn up myself.
        $endgroup$
        – neo
        May 23 at 15:02










      • $begingroup$
        The Thunder .50 is essentially a hand cannon: it's got all the usual features of an artillery piece (pneumatic recoil absorption, muzzle brake, etc.) in a nominally hand-held form factor.
        $endgroup$
        – Mark
        May 23 at 20:57






      • 1




        $begingroup$
        @Mark, it is also hilarious to watch someone fire it, you can almost see their body screaming NOPE afterwards. And this has much less recoil than a 22mm would have.
        $endgroup$
        – John
        May 23 at 23:49






      • 1




        $begingroup$
        There have historically been problems using necked cartridges in revolver. Case expansion (specifically in the shoulder) sets the cartridge back, and then stays that way, locking up the cylinder. Yes, necked case revolvers have been made to work, but it require scrupulous cleaning and oil removal inside the chambers. If you need to reload two or three times in combat, you're virtually certain to run into this issue.
        $endgroup$
        – Zeiss Ikon
        May 24 at 11:23










      • $begingroup$
        I suspect Hellboy's ammo is just fine, and it has the large caliber because it isn't just a slug. It's like how modern fighter jets really struggle to have ammo smaller than 25mm... the bullets are tiny artillery shells with their own HE, shrapnel and fuzes (yes Z). In fact fighter-jet "training blanks" actually are plain slugs, so still pretty deadly. Hellboy's gun only needs to get the bullet there, the bullet itself does all the heavy lifting...
        $endgroup$
        – Harper
        May 26 at 15:24














      25














      25










      25







      $begingroup$

      Yes



      28mm revolvers exist they are just not practical weapons (~14lbs).
      a 22mm would be almost identical to a 8guage shotgun shell, except it will have more powder.



      enter image description here



      Hellboy's gun has relatively low power you don't want to copy it.
      The length of the cartridge also matters as much as the diameter, a wide short cartridge is not going to pack much punch. Hellboy's cartridges are wide but very short, I would not expect more punch than a large pistol or mid range shotgun.



      Your homebrewed cartridges are a bit better, almost identical to a 8 gauge shotgun shell (21.21mm). Homemade 12gauge shotgun revolvers exist and weight around 11lbs, you can drop some weight by using better materials. note your real problem is you have a lot of wasted metal making a cylinder big enough for 6 large diameter rounds, the cylinder needs to be wide enough to fit them all at equal distance which makes for a very large diameter cylinder. That is a lot of wasted metal in the center of the cylinder, and that adds a lot of weight. That is why the largest handguns often only hold 5 rounds, it has a huge impact on weight.



      enter image description here



      But you could make your bullets a lot better.



      So how do you really take advantage of augmentation, use a necked round, you can drastically improve the power for similar recoil. What extreme can you get wit that. well the largest common rifle round is the .50bmg, is 12.7 X 99mm and packs significantly more punch than a an 8 gauge shotgun will.



      A .50 bmg handgun also exists (see image) .50 bmg is necked rifle round and produces 65,000 psi (450,000 kPa) and 2-3000 foot pounds of force so a measly 1200 is no problem. Even hand made .50bmg revolvers exist but are comically heavy. A fireable single shot .50 bmg production handgun also exists, THUNDER 50 BMG, it weights 12lbs empty although a significant amount of its mass is recoil absorption.



      enter image description here



      So in the end you are looking at something in the 10-13 lb range. could a normal human fire it without aid, no, not in any way reliable or combat useful. But an augmented human likely could you just need something to absorb recoil and carry the weight.






      share|improve this answer











      $endgroup$



      Yes



      28mm revolvers exist they are just not practical weapons (~14lbs).
      a 22mm would be almost identical to a 8guage shotgun shell, except it will have more powder.



      enter image description here



      Hellboy's gun has relatively low power you don't want to copy it.
      The length of the cartridge also matters as much as the diameter, a wide short cartridge is not going to pack much punch. Hellboy's cartridges are wide but very short, I would not expect more punch than a large pistol or mid range shotgun.



      Your homebrewed cartridges are a bit better, almost identical to a 8 gauge shotgun shell (21.21mm). Homemade 12gauge shotgun revolvers exist and weight around 11lbs, you can drop some weight by using better materials. note your real problem is you have a lot of wasted metal making a cylinder big enough for 6 large diameter rounds, the cylinder needs to be wide enough to fit them all at equal distance which makes for a very large diameter cylinder. That is a lot of wasted metal in the center of the cylinder, and that adds a lot of weight. That is why the largest handguns often only hold 5 rounds, it has a huge impact on weight.



      enter image description here



      But you could make your bullets a lot better.



      So how do you really take advantage of augmentation, use a necked round, you can drastically improve the power for similar recoil. What extreme can you get wit that. well the largest common rifle round is the .50bmg, is 12.7 X 99mm and packs significantly more punch than a an 8 gauge shotgun will.



      A .50 bmg handgun also exists (see image) .50 bmg is necked rifle round and produces 65,000 psi (450,000 kPa) and 2-3000 foot pounds of force so a measly 1200 is no problem. Even hand made .50bmg revolvers exist but are comically heavy. A fireable single shot .50 bmg production handgun also exists, THUNDER 50 BMG, it weights 12lbs empty although a significant amount of its mass is recoil absorption.



      enter image description here



      So in the end you are looking at something in the 10-13 lb range. could a normal human fire it without aid, no, not in any way reliable or combat useful. But an augmented human likely could you just need something to absorb recoil and carry the weight.







      share|improve this answer














      share|improve this answer



      share|improve this answer








      edited May 24 at 0:58

























      answered May 23 at 14:59









      JohnJohn

      42.7k11 gold badges61 silver badges144 bronze badges




      42.7k11 gold badges61 silver badges144 bronze badges















      • $begingroup$
        Thanks, those are some interesting examples I was unable to turn up myself.
        $endgroup$
        – neo
        May 23 at 15:02










      • $begingroup$
        The Thunder .50 is essentially a hand cannon: it's got all the usual features of an artillery piece (pneumatic recoil absorption, muzzle brake, etc.) in a nominally hand-held form factor.
        $endgroup$
        – Mark
        May 23 at 20:57






      • 1




        $begingroup$
        @Mark, it is also hilarious to watch someone fire it, you can almost see their body screaming NOPE afterwards. And this has much less recoil than a 22mm would have.
        $endgroup$
        – John
        May 23 at 23:49






      • 1




        $begingroup$
        There have historically been problems using necked cartridges in revolver. Case expansion (specifically in the shoulder) sets the cartridge back, and then stays that way, locking up the cylinder. Yes, necked case revolvers have been made to work, but it require scrupulous cleaning and oil removal inside the chambers. If you need to reload two or three times in combat, you're virtually certain to run into this issue.
        $endgroup$
        – Zeiss Ikon
        May 24 at 11:23










      • $begingroup$
        I suspect Hellboy's ammo is just fine, and it has the large caliber because it isn't just a slug. It's like how modern fighter jets really struggle to have ammo smaller than 25mm... the bullets are tiny artillery shells with their own HE, shrapnel and fuzes (yes Z). In fact fighter-jet "training blanks" actually are plain slugs, so still pretty deadly. Hellboy's gun only needs to get the bullet there, the bullet itself does all the heavy lifting...
        $endgroup$
        – Harper
        May 26 at 15:24


















      • $begingroup$
        Thanks, those are some interesting examples I was unable to turn up myself.
        $endgroup$
        – neo
        May 23 at 15:02










      • $begingroup$
        The Thunder .50 is essentially a hand cannon: it's got all the usual features of an artillery piece (pneumatic recoil absorption, muzzle brake, etc.) in a nominally hand-held form factor.
        $endgroup$
        – Mark
        May 23 at 20:57






      • 1




        $begingroup$
        @Mark, it is also hilarious to watch someone fire it, you can almost see their body screaming NOPE afterwards. And this has much less recoil than a 22mm would have.
        $endgroup$
        – John
        May 23 at 23:49






      • 1




        $begingroup$
        There have historically been problems using necked cartridges in revolver. Case expansion (specifically in the shoulder) sets the cartridge back, and then stays that way, locking up the cylinder. Yes, necked case revolvers have been made to work, but it require scrupulous cleaning and oil removal inside the chambers. If you need to reload two or three times in combat, you're virtually certain to run into this issue.
        $endgroup$
        – Zeiss Ikon
        May 24 at 11:23










      • $begingroup$
        I suspect Hellboy's ammo is just fine, and it has the large caliber because it isn't just a slug. It's like how modern fighter jets really struggle to have ammo smaller than 25mm... the bullets are tiny artillery shells with their own HE, shrapnel and fuzes (yes Z). In fact fighter-jet "training blanks" actually are plain slugs, so still pretty deadly. Hellboy's gun only needs to get the bullet there, the bullet itself does all the heavy lifting...
        $endgroup$
        – Harper
        May 26 at 15:24
















      $begingroup$
      Thanks, those are some interesting examples I was unable to turn up myself.
      $endgroup$
      – neo
      May 23 at 15:02




      $begingroup$
      Thanks, those are some interesting examples I was unable to turn up myself.
      $endgroup$
      – neo
      May 23 at 15:02












      $begingroup$
      The Thunder .50 is essentially a hand cannon: it's got all the usual features of an artillery piece (pneumatic recoil absorption, muzzle brake, etc.) in a nominally hand-held form factor.
      $endgroup$
      – Mark
      May 23 at 20:57




      $begingroup$
      The Thunder .50 is essentially a hand cannon: it's got all the usual features of an artillery piece (pneumatic recoil absorption, muzzle brake, etc.) in a nominally hand-held form factor.
      $endgroup$
      – Mark
      May 23 at 20:57




      1




      1




      $begingroup$
      @Mark, it is also hilarious to watch someone fire it, you can almost see their body screaming NOPE afterwards. And this has much less recoil than a 22mm would have.
      $endgroup$
      – John
      May 23 at 23:49




      $begingroup$
      @Mark, it is also hilarious to watch someone fire it, you can almost see their body screaming NOPE afterwards. And this has much less recoil than a 22mm would have.
      $endgroup$
      – John
      May 23 at 23:49




      1




      1




      $begingroup$
      There have historically been problems using necked cartridges in revolver. Case expansion (specifically in the shoulder) sets the cartridge back, and then stays that way, locking up the cylinder. Yes, necked case revolvers have been made to work, but it require scrupulous cleaning and oil removal inside the chambers. If you need to reload two or three times in combat, you're virtually certain to run into this issue.
      $endgroup$
      – Zeiss Ikon
      May 24 at 11:23




      $begingroup$
      There have historically been problems using necked cartridges in revolver. Case expansion (specifically in the shoulder) sets the cartridge back, and then stays that way, locking up the cylinder. Yes, necked case revolvers have been made to work, but it require scrupulous cleaning and oil removal inside the chambers. If you need to reload two or three times in combat, you're virtually certain to run into this issue.
      $endgroup$
      – Zeiss Ikon
      May 24 at 11:23












      $begingroup$
      I suspect Hellboy's ammo is just fine, and it has the large caliber because it isn't just a slug. It's like how modern fighter jets really struggle to have ammo smaller than 25mm... the bullets are tiny artillery shells with their own HE, shrapnel and fuzes (yes Z). In fact fighter-jet "training blanks" actually are plain slugs, so still pretty deadly. Hellboy's gun only needs to get the bullet there, the bullet itself does all the heavy lifting...
      $endgroup$
      – Harper
      May 26 at 15:24




      $begingroup$
      I suspect Hellboy's ammo is just fine, and it has the large caliber because it isn't just a slug. It's like how modern fighter jets really struggle to have ammo smaller than 25mm... the bullets are tiny artillery shells with their own HE, shrapnel and fuzes (yes Z). In fact fighter-jet "training blanks" actually are plain slugs, so still pretty deadly. Hellboy's gun only needs to get the bullet there, the bullet itself does all the heavy lifting...
      $endgroup$
      – Harper
      May 26 at 15:24











      8














      $begingroup$

      You Betcha



      If we temporarily ignore the capacity of the wielder to carry the monster, you could weld a hand grip and stock to a Howitzer. Maybe only the Hulk could use it. But it would work just dandy.



      So, having proven the premise is possible, let's focus on plausible. I give you:



      Recoilless rifles




      A recoilless rifle, recoilless launcher or recoilless gun, sometimes abbreviated "RR" or "RCL" (for ReCoilLess) is a type of lightweight artillery system or man-portable launcher that is designed to eject some form of countermass such as propellant gas from the rear of the weapon at the moment of firing, creating forward thrust that counteracts most of the weapon's recoil.




      Now, I know what you're thinking. "These are usually shoulder-mounted critters like anti-tank rockets because, well, the back-blast might cause mild skin irritation on the user's face." And you'd be completely correct. But I'm thinking that all we need is some kind of shield, right? Let's think Logan's Run.



      Copyright MGM



      copyright MGM



      Having done it's work, the back-blast is directed harmlessly out of the user's way. Just because it has the potential to start forest fires shouldn't detract from the natural beauty and form of the weapon. Besides, who doesn't like a picture of Jenny Agutter?



      I'm going to leave the ability of the user to hold the weight of the weapon up to the OP. Frankly, I'd hate to hold it on some Evil Genius who's decided to start monologing. No matter how strong you are, that much weight would become uncomfortable mighty quickly. Isometrics, anyone?



      Edit:  Daniel B brings up a good point. There's an out-of-manufacture class of weapons that could solve most if not all of your problems: the Gyrojet.




      The Gyrojet is a family of unique firearms developed in the 1960s named for the method of gyroscopically stabilizing its projectiles. Rather than inert bullets, Gyrojets fire small rockets called Microjets which have little recoil and do not require a heavy barrel or chamber to resist the pressure of the combustion gases. Velocity on leaving the tube was very low, but increased to around 1,250 feet per second (380 m/s) at 30 feet (9.1 m). The result is a very lightweight weapon.




      The Gyrojet is the realization of considerable sci-fi fodder, most notably from the Buck Rogers series of comics (1933) and books where the "rocket pistol" was a staple. I'm especially fond of my copy of Philip Francis Nowlan's Amargeddon 2419 A.D.: The Seminal "Buck Rogers" Novel,1 (1978) which has a parallel universe, disintergrator rays, and repulsar beams — and doesn't waste one darn minute trying to explain any of them. Oh, yeah....





      1Rendered nearly worthless to anyone but me due to the number of times I've read the book. Yes, I understand there's value in mint condition books — but the true value of a book is discovered by how worn the cover and pages are. Who cares about a book that's never read?






      share|improve this answer











      $endgroup$











      • 2




        $begingroup$
        who doesn't like a picture of Jenny Agutter Tempting to vote you up just for that remark, but I'm pretty sure it's against the spirit of the rules. :-)
        $endgroup$
        – StephenG
        May 23 at 19:35






      • 1




        $begingroup$
        @JBH: I was going to put an answer involving gyrojets but this answer covers the basic concept without actually naming them. Could you edit in a reference to them?
        $endgroup$
        – Daniel B
        May 24 at 0:20






      • 2




        $begingroup$
        I wish I could upvote twice for the "Just because it has the potential to start forest fires should detract from the natural beauty and form of the weapon." part.
        $endgroup$
        – Renan
        May 24 at 1:44










      • $begingroup$
        @DanielB Done! It actually reminded me of a favorite novel, which I had the chance to mention. Thanks!
        $endgroup$
        – JBH
        May 24 at 4:15










      • $begingroup$
        "Having done it's work, the back-blast is directed harmlessly out of the user's way" and onto the people standing next to him. Or is that what you meant by "Just because it has the potential to start forest fires shouldn't detract from the natural beauty and form of the weapon"?
        $endgroup$
        – RonJohn
        May 27 at 5:22
















      8














      $begingroup$

      You Betcha



      If we temporarily ignore the capacity of the wielder to carry the monster, you could weld a hand grip and stock to a Howitzer. Maybe only the Hulk could use it. But it would work just dandy.



      So, having proven the premise is possible, let's focus on plausible. I give you:



      Recoilless rifles




      A recoilless rifle, recoilless launcher or recoilless gun, sometimes abbreviated "RR" or "RCL" (for ReCoilLess) is a type of lightweight artillery system or man-portable launcher that is designed to eject some form of countermass such as propellant gas from the rear of the weapon at the moment of firing, creating forward thrust that counteracts most of the weapon's recoil.




      Now, I know what you're thinking. "These are usually shoulder-mounted critters like anti-tank rockets because, well, the back-blast might cause mild skin irritation on the user's face." And you'd be completely correct. But I'm thinking that all we need is some kind of shield, right? Let's think Logan's Run.



      Copyright MGM



      copyright MGM



      Having done it's work, the back-blast is directed harmlessly out of the user's way. Just because it has the potential to start forest fires shouldn't detract from the natural beauty and form of the weapon. Besides, who doesn't like a picture of Jenny Agutter?



      I'm going to leave the ability of the user to hold the weight of the weapon up to the OP. Frankly, I'd hate to hold it on some Evil Genius who's decided to start monologing. No matter how strong you are, that much weight would become uncomfortable mighty quickly. Isometrics, anyone?



      Edit:  Daniel B brings up a good point. There's an out-of-manufacture class of weapons that could solve most if not all of your problems: the Gyrojet.




      The Gyrojet is a family of unique firearms developed in the 1960s named for the method of gyroscopically stabilizing its projectiles. Rather than inert bullets, Gyrojets fire small rockets called Microjets which have little recoil and do not require a heavy barrel or chamber to resist the pressure of the combustion gases. Velocity on leaving the tube was very low, but increased to around 1,250 feet per second (380 m/s) at 30 feet (9.1 m). The result is a very lightweight weapon.




      The Gyrojet is the realization of considerable sci-fi fodder, most notably from the Buck Rogers series of comics (1933) and books where the "rocket pistol" was a staple. I'm especially fond of my copy of Philip Francis Nowlan's Amargeddon 2419 A.D.: The Seminal "Buck Rogers" Novel,1 (1978) which has a parallel universe, disintergrator rays, and repulsar beams — and doesn't waste one darn minute trying to explain any of them. Oh, yeah....





      1Rendered nearly worthless to anyone but me due to the number of times I've read the book. Yes, I understand there's value in mint condition books — but the true value of a book is discovered by how worn the cover and pages are. Who cares about a book that's never read?






      share|improve this answer











      $endgroup$











      • 2




        $begingroup$
        who doesn't like a picture of Jenny Agutter Tempting to vote you up just for that remark, but I'm pretty sure it's against the spirit of the rules. :-)
        $endgroup$
        – StephenG
        May 23 at 19:35






      • 1




        $begingroup$
        @JBH: I was going to put an answer involving gyrojets but this answer covers the basic concept without actually naming them. Could you edit in a reference to them?
        $endgroup$
        – Daniel B
        May 24 at 0:20






      • 2




        $begingroup$
        I wish I could upvote twice for the "Just because it has the potential to start forest fires should detract from the natural beauty and form of the weapon." part.
        $endgroup$
        – Renan
        May 24 at 1:44










      • $begingroup$
        @DanielB Done! It actually reminded me of a favorite novel, which I had the chance to mention. Thanks!
        $endgroup$
        – JBH
        May 24 at 4:15










      • $begingroup$
        "Having done it's work, the back-blast is directed harmlessly out of the user's way" and onto the people standing next to him. Or is that what you meant by "Just because it has the potential to start forest fires shouldn't detract from the natural beauty and form of the weapon"?
        $endgroup$
        – RonJohn
        May 27 at 5:22














      8














      8










      8







      $begingroup$

      You Betcha



      If we temporarily ignore the capacity of the wielder to carry the monster, you could weld a hand grip and stock to a Howitzer. Maybe only the Hulk could use it. But it would work just dandy.



      So, having proven the premise is possible, let's focus on plausible. I give you:



      Recoilless rifles




      A recoilless rifle, recoilless launcher or recoilless gun, sometimes abbreviated "RR" or "RCL" (for ReCoilLess) is a type of lightweight artillery system or man-portable launcher that is designed to eject some form of countermass such as propellant gas from the rear of the weapon at the moment of firing, creating forward thrust that counteracts most of the weapon's recoil.




      Now, I know what you're thinking. "These are usually shoulder-mounted critters like anti-tank rockets because, well, the back-blast might cause mild skin irritation on the user's face." And you'd be completely correct. But I'm thinking that all we need is some kind of shield, right? Let's think Logan's Run.



      Copyright MGM



      copyright MGM



      Having done it's work, the back-blast is directed harmlessly out of the user's way. Just because it has the potential to start forest fires shouldn't detract from the natural beauty and form of the weapon. Besides, who doesn't like a picture of Jenny Agutter?



      I'm going to leave the ability of the user to hold the weight of the weapon up to the OP. Frankly, I'd hate to hold it on some Evil Genius who's decided to start monologing. No matter how strong you are, that much weight would become uncomfortable mighty quickly. Isometrics, anyone?



      Edit:  Daniel B brings up a good point. There's an out-of-manufacture class of weapons that could solve most if not all of your problems: the Gyrojet.




      The Gyrojet is a family of unique firearms developed in the 1960s named for the method of gyroscopically stabilizing its projectiles. Rather than inert bullets, Gyrojets fire small rockets called Microjets which have little recoil and do not require a heavy barrel or chamber to resist the pressure of the combustion gases. Velocity on leaving the tube was very low, but increased to around 1,250 feet per second (380 m/s) at 30 feet (9.1 m). The result is a very lightweight weapon.




      The Gyrojet is the realization of considerable sci-fi fodder, most notably from the Buck Rogers series of comics (1933) and books where the "rocket pistol" was a staple. I'm especially fond of my copy of Philip Francis Nowlan's Amargeddon 2419 A.D.: The Seminal "Buck Rogers" Novel,1 (1978) which has a parallel universe, disintergrator rays, and repulsar beams — and doesn't waste one darn minute trying to explain any of them. Oh, yeah....





      1Rendered nearly worthless to anyone but me due to the number of times I've read the book. Yes, I understand there's value in mint condition books — but the true value of a book is discovered by how worn the cover and pages are. Who cares about a book that's never read?






      share|improve this answer











      $endgroup$



      You Betcha



      If we temporarily ignore the capacity of the wielder to carry the monster, you could weld a hand grip and stock to a Howitzer. Maybe only the Hulk could use it. But it would work just dandy.



      So, having proven the premise is possible, let's focus on plausible. I give you:



      Recoilless rifles




      A recoilless rifle, recoilless launcher or recoilless gun, sometimes abbreviated "RR" or "RCL" (for ReCoilLess) is a type of lightweight artillery system or man-portable launcher that is designed to eject some form of countermass such as propellant gas from the rear of the weapon at the moment of firing, creating forward thrust that counteracts most of the weapon's recoil.




      Now, I know what you're thinking. "These are usually shoulder-mounted critters like anti-tank rockets because, well, the back-blast might cause mild skin irritation on the user's face." And you'd be completely correct. But I'm thinking that all we need is some kind of shield, right? Let's think Logan's Run.



      Copyright MGM



      copyright MGM



      Having done it's work, the back-blast is directed harmlessly out of the user's way. Just because it has the potential to start forest fires shouldn't detract from the natural beauty and form of the weapon. Besides, who doesn't like a picture of Jenny Agutter?



      I'm going to leave the ability of the user to hold the weight of the weapon up to the OP. Frankly, I'd hate to hold it on some Evil Genius who's decided to start monologing. No matter how strong you are, that much weight would become uncomfortable mighty quickly. Isometrics, anyone?



      Edit:  Daniel B brings up a good point. There's an out-of-manufacture class of weapons that could solve most if not all of your problems: the Gyrojet.




      The Gyrojet is a family of unique firearms developed in the 1960s named for the method of gyroscopically stabilizing its projectiles. Rather than inert bullets, Gyrojets fire small rockets called Microjets which have little recoil and do not require a heavy barrel or chamber to resist the pressure of the combustion gases. Velocity on leaving the tube was very low, but increased to around 1,250 feet per second (380 m/s) at 30 feet (9.1 m). The result is a very lightweight weapon.




      The Gyrojet is the realization of considerable sci-fi fodder, most notably from the Buck Rogers series of comics (1933) and books where the "rocket pistol" was a staple. I'm especially fond of my copy of Philip Francis Nowlan's Amargeddon 2419 A.D.: The Seminal "Buck Rogers" Novel,1 (1978) which has a parallel universe, disintergrator rays, and repulsar beams — and doesn't waste one darn minute trying to explain any of them. Oh, yeah....





      1Rendered nearly worthless to anyone but me due to the number of times I've read the book. Yes, I understand there's value in mint condition books — but the true value of a book is discovered by how worn the cover and pages are. Who cares about a book that's never read?







      share|improve this answer














      share|improve this answer



      share|improve this answer








      edited May 24 at 4:14

























      answered May 23 at 18:31









      JBHJBH

      57.2k9 gold badges131 silver badges276 bronze badges




      57.2k9 gold badges131 silver badges276 bronze badges











      • 2




        $begingroup$
        who doesn't like a picture of Jenny Agutter Tempting to vote you up just for that remark, but I'm pretty sure it's against the spirit of the rules. :-)
        $endgroup$
        – StephenG
        May 23 at 19:35






      • 1




        $begingroup$
        @JBH: I was going to put an answer involving gyrojets but this answer covers the basic concept without actually naming them. Could you edit in a reference to them?
        $endgroup$
        – Daniel B
        May 24 at 0:20






      • 2




        $begingroup$
        I wish I could upvote twice for the "Just because it has the potential to start forest fires should detract from the natural beauty and form of the weapon." part.
        $endgroup$
        – Renan
        May 24 at 1:44










      • $begingroup$
        @DanielB Done! It actually reminded me of a favorite novel, which I had the chance to mention. Thanks!
        $endgroup$
        – JBH
        May 24 at 4:15










      • $begingroup$
        "Having done it's work, the back-blast is directed harmlessly out of the user's way" and onto the people standing next to him. Or is that what you meant by "Just because it has the potential to start forest fires shouldn't detract from the natural beauty and form of the weapon"?
        $endgroup$
        – RonJohn
        May 27 at 5:22














      • 2




        $begingroup$
        who doesn't like a picture of Jenny Agutter Tempting to vote you up just for that remark, but I'm pretty sure it's against the spirit of the rules. :-)
        $endgroup$
        – StephenG
        May 23 at 19:35






      • 1




        $begingroup$
        @JBH: I was going to put an answer involving gyrojets but this answer covers the basic concept without actually naming them. Could you edit in a reference to them?
        $endgroup$
        – Daniel B
        May 24 at 0:20






      • 2




        $begingroup$
        I wish I could upvote twice for the "Just because it has the potential to start forest fires should detract from the natural beauty and form of the weapon." part.
        $endgroup$
        – Renan
        May 24 at 1:44










      • $begingroup$
        @DanielB Done! It actually reminded me of a favorite novel, which I had the chance to mention. Thanks!
        $endgroup$
        – JBH
        May 24 at 4:15










      • $begingroup$
        "Having done it's work, the back-blast is directed harmlessly out of the user's way" and onto the people standing next to him. Or is that what you meant by "Just because it has the potential to start forest fires shouldn't detract from the natural beauty and form of the weapon"?
        $endgroup$
        – RonJohn
        May 27 at 5:22








      2




      2




      $begingroup$
      who doesn't like a picture of Jenny Agutter Tempting to vote you up just for that remark, but I'm pretty sure it's against the spirit of the rules. :-)
      $endgroup$
      – StephenG
      May 23 at 19:35




      $begingroup$
      who doesn't like a picture of Jenny Agutter Tempting to vote you up just for that remark, but I'm pretty sure it's against the spirit of the rules. :-)
      $endgroup$
      – StephenG
      May 23 at 19:35




      1




      1




      $begingroup$
      @JBH: I was going to put an answer involving gyrojets but this answer covers the basic concept without actually naming them. Could you edit in a reference to them?
      $endgroup$
      – Daniel B
      May 24 at 0:20




      $begingroup$
      @JBH: I was going to put an answer involving gyrojets but this answer covers the basic concept without actually naming them. Could you edit in a reference to them?
      $endgroup$
      – Daniel B
      May 24 at 0:20




      2




      2




      $begingroup$
      I wish I could upvote twice for the "Just because it has the potential to start forest fires should detract from the natural beauty and form of the weapon." part.
      $endgroup$
      – Renan
      May 24 at 1:44




      $begingroup$
      I wish I could upvote twice for the "Just because it has the potential to start forest fires should detract from the natural beauty and form of the weapon." part.
      $endgroup$
      – Renan
      May 24 at 1:44












      $begingroup$
      @DanielB Done! It actually reminded me of a favorite novel, which I had the chance to mention. Thanks!
      $endgroup$
      – JBH
      May 24 at 4:15




      $begingroup$
      @DanielB Done! It actually reminded me of a favorite novel, which I had the chance to mention. Thanks!
      $endgroup$
      – JBH
      May 24 at 4:15












      $begingroup$
      "Having done it's work, the back-blast is directed harmlessly out of the user's way" and onto the people standing next to him. Or is that what you meant by "Just because it has the potential to start forest fires shouldn't detract from the natural beauty and form of the weapon"?
      $endgroup$
      – RonJohn
      May 27 at 5:22




      $begingroup$
      "Having done it's work, the back-blast is directed harmlessly out of the user's way" and onto the people standing next to him. Or is that what you meant by "Just because it has the potential to start forest fires shouldn't detract from the natural beauty and form of the weapon"?
      $endgroup$
      – RonJohn
      May 27 at 5:22











      8














      $begingroup$

      I've crunched the numbers a bit, and I have to admit that both variants of the weapon are possible, although very unwieldy. What I don't like here is a narrative part - I just can't imagine someone developing such a revolver as a battlefield weapon.



      So I've thought about some corrections. Say, we have an increase in human augmentation, robotics, etc. So there are targets that is very hard to take down. I can imagine a government agency developing a ".758 BMG" necked up wildcat of .50 BMG to deal with heavily armored augmented threats in urban environment. The main weapon it used in is a Barret-type rifle, bullpup, suppressed. The round is subsonic and, perhaps, frangible. It has a muzzle energy of about 14000 joules and delivers most of them to the target without overpenetration. It can be fired by non-augmented human operator as an antimateriel rifle, and by a non-augmented operator standing up. .758 BMG rifle is absolutely valid and can be a normal item in your world.



      .758 revolver is an experimental crazy offshoot of this technology. It was envisioned as a sidearm for augmented operators, firing the same round. I'm not sure this weapon was ever accepted and mass-produced. It's just too crazy and unwieldy, with too short a barrel for the round it fires. But there can be a number of prototype examples.



      For unaugmented human to use it, it needs severely underpowered rounds. And, to be honest, it sucks. Something like .50 Beowulf carbine would give you about the same power (<3000 joules) in much more comfortable and ergonomic package. And all those carbines are of themselves a compromise in trying to fit a big caliber bullet in short action. So it is very much a novelty gimmick and should be treated as such.



      Another part that I don't like is that too underload this cartridge you need to leave A LOT of space in it empty. I'm not sure it is safe. My proposed solution was to make the original .758 cartridge caseless. Without brass it would be shorter - maybe around 3.5 inches as opposed to .50 BMG 5 inches. Then the character would be able to manufacture his own weaker cartridges with lighter bullets from cut down .50 BMG brass.



      UPD: I kinda forgot your full-power round was supersonic, but IMO, when you go as heavy as 3000 grains, you can just make it subsonic. I envisioned ".758 Barret rifle" as a high-powered analogue to Russian 9x39 rifles, with heavy rounds, subsonic and suppressed, for urban usage.






      share|improve this answer











      $endgroup$















      • $begingroup$
        Yes! That's exactly it. A rifle does exist and some nut made a handgun with it's cartridge. The government made a limited run of 100 or in my story. To give as decoration to officers. And you gave me an idea. The original prototype could fire the human safe round and the officer's the more powerful full bullet. Since to be an officer you just about have to be an aug. This gives me two versions, which I wanted to avoid, but it does make more sense. I have to admit you are the only one who has made me seriously consider just not giving it to my Character if they don't have augments.
        $endgroup$
        – neo
        May 24 at 15:18
















      8














      $begingroup$

      I've crunched the numbers a bit, and I have to admit that both variants of the weapon are possible, although very unwieldy. What I don't like here is a narrative part - I just can't imagine someone developing such a revolver as a battlefield weapon.



      So I've thought about some corrections. Say, we have an increase in human augmentation, robotics, etc. So there are targets that is very hard to take down. I can imagine a government agency developing a ".758 BMG" necked up wildcat of .50 BMG to deal with heavily armored augmented threats in urban environment. The main weapon it used in is a Barret-type rifle, bullpup, suppressed. The round is subsonic and, perhaps, frangible. It has a muzzle energy of about 14000 joules and delivers most of them to the target without overpenetration. It can be fired by non-augmented human operator as an antimateriel rifle, and by a non-augmented operator standing up. .758 BMG rifle is absolutely valid and can be a normal item in your world.



      .758 revolver is an experimental crazy offshoot of this technology. It was envisioned as a sidearm for augmented operators, firing the same round. I'm not sure this weapon was ever accepted and mass-produced. It's just too crazy and unwieldy, with too short a barrel for the round it fires. But there can be a number of prototype examples.



      For unaugmented human to use it, it needs severely underpowered rounds. And, to be honest, it sucks. Something like .50 Beowulf carbine would give you about the same power (<3000 joules) in much more comfortable and ergonomic package. And all those carbines are of themselves a compromise in trying to fit a big caliber bullet in short action. So it is very much a novelty gimmick and should be treated as such.



      Another part that I don't like is that too underload this cartridge you need to leave A LOT of space in it empty. I'm not sure it is safe. My proposed solution was to make the original .758 cartridge caseless. Without brass it would be shorter - maybe around 3.5 inches as opposed to .50 BMG 5 inches. Then the character would be able to manufacture his own weaker cartridges with lighter bullets from cut down .50 BMG brass.



      UPD: I kinda forgot your full-power round was supersonic, but IMO, when you go as heavy as 3000 grains, you can just make it subsonic. I envisioned ".758 Barret rifle" as a high-powered analogue to Russian 9x39 rifles, with heavy rounds, subsonic and suppressed, for urban usage.






      share|improve this answer











      $endgroup$















      • $begingroup$
        Yes! That's exactly it. A rifle does exist and some nut made a handgun with it's cartridge. The government made a limited run of 100 or in my story. To give as decoration to officers. And you gave me an idea. The original prototype could fire the human safe round and the officer's the more powerful full bullet. Since to be an officer you just about have to be an aug. This gives me two versions, which I wanted to avoid, but it does make more sense. I have to admit you are the only one who has made me seriously consider just not giving it to my Character if they don't have augments.
        $endgroup$
        – neo
        May 24 at 15:18














      8














      8










      8







      $begingroup$

      I've crunched the numbers a bit, and I have to admit that both variants of the weapon are possible, although very unwieldy. What I don't like here is a narrative part - I just can't imagine someone developing such a revolver as a battlefield weapon.



      So I've thought about some corrections. Say, we have an increase in human augmentation, robotics, etc. So there are targets that is very hard to take down. I can imagine a government agency developing a ".758 BMG" necked up wildcat of .50 BMG to deal with heavily armored augmented threats in urban environment. The main weapon it used in is a Barret-type rifle, bullpup, suppressed. The round is subsonic and, perhaps, frangible. It has a muzzle energy of about 14000 joules and delivers most of them to the target without overpenetration. It can be fired by non-augmented human operator as an antimateriel rifle, and by a non-augmented operator standing up. .758 BMG rifle is absolutely valid and can be a normal item in your world.



      .758 revolver is an experimental crazy offshoot of this technology. It was envisioned as a sidearm for augmented operators, firing the same round. I'm not sure this weapon was ever accepted and mass-produced. It's just too crazy and unwieldy, with too short a barrel for the round it fires. But there can be a number of prototype examples.



      For unaugmented human to use it, it needs severely underpowered rounds. And, to be honest, it sucks. Something like .50 Beowulf carbine would give you about the same power (<3000 joules) in much more comfortable and ergonomic package. And all those carbines are of themselves a compromise in trying to fit a big caliber bullet in short action. So it is very much a novelty gimmick and should be treated as such.



      Another part that I don't like is that too underload this cartridge you need to leave A LOT of space in it empty. I'm not sure it is safe. My proposed solution was to make the original .758 cartridge caseless. Without brass it would be shorter - maybe around 3.5 inches as opposed to .50 BMG 5 inches. Then the character would be able to manufacture his own weaker cartridges with lighter bullets from cut down .50 BMG brass.



      UPD: I kinda forgot your full-power round was supersonic, but IMO, when you go as heavy as 3000 grains, you can just make it subsonic. I envisioned ".758 Barret rifle" as a high-powered analogue to Russian 9x39 rifles, with heavy rounds, subsonic and suppressed, for urban usage.






      share|improve this answer











      $endgroup$



      I've crunched the numbers a bit, and I have to admit that both variants of the weapon are possible, although very unwieldy. What I don't like here is a narrative part - I just can't imagine someone developing such a revolver as a battlefield weapon.



      So I've thought about some corrections. Say, we have an increase in human augmentation, robotics, etc. So there are targets that is very hard to take down. I can imagine a government agency developing a ".758 BMG" necked up wildcat of .50 BMG to deal with heavily armored augmented threats in urban environment. The main weapon it used in is a Barret-type rifle, bullpup, suppressed. The round is subsonic and, perhaps, frangible. It has a muzzle energy of about 14000 joules and delivers most of them to the target without overpenetration. It can be fired by non-augmented human operator as an antimateriel rifle, and by a non-augmented operator standing up. .758 BMG rifle is absolutely valid and can be a normal item in your world.



      .758 revolver is an experimental crazy offshoot of this technology. It was envisioned as a sidearm for augmented operators, firing the same round. I'm not sure this weapon was ever accepted and mass-produced. It's just too crazy and unwieldy, with too short a barrel for the round it fires. But there can be a number of prototype examples.



      For unaugmented human to use it, it needs severely underpowered rounds. And, to be honest, it sucks. Something like .50 Beowulf carbine would give you about the same power (<3000 joules) in much more comfortable and ergonomic package. And all those carbines are of themselves a compromise in trying to fit a big caliber bullet in short action. So it is very much a novelty gimmick and should be treated as such.



      Another part that I don't like is that too underload this cartridge you need to leave A LOT of space in it empty. I'm not sure it is safe. My proposed solution was to make the original .758 cartridge caseless. Without brass it would be shorter - maybe around 3.5 inches as opposed to .50 BMG 5 inches. Then the character would be able to manufacture his own weaker cartridges with lighter bullets from cut down .50 BMG brass.



      UPD: I kinda forgot your full-power round was supersonic, but IMO, when you go as heavy as 3000 grains, you can just make it subsonic. I envisioned ".758 Barret rifle" as a high-powered analogue to Russian 9x39 rifles, with heavy rounds, subsonic and suppressed, for urban usage.







      share|improve this answer














      share|improve this answer



      share|improve this answer








      edited May 24 at 9:02

























      answered May 23 at 17:14









      CumehtarCumehtar

      4,3837 silver badges30 bronze badges




      4,3837 silver badges30 bronze badges















      • $begingroup$
        Yes! That's exactly it. A rifle does exist and some nut made a handgun with it's cartridge. The government made a limited run of 100 or in my story. To give as decoration to officers. And you gave me an idea. The original prototype could fire the human safe round and the officer's the more powerful full bullet. Since to be an officer you just about have to be an aug. This gives me two versions, which I wanted to avoid, but it does make more sense. I have to admit you are the only one who has made me seriously consider just not giving it to my Character if they don't have augments.
        $endgroup$
        – neo
        May 24 at 15:18


















      • $begingroup$
        Yes! That's exactly it. A rifle does exist and some nut made a handgun with it's cartridge. The government made a limited run of 100 or in my story. To give as decoration to officers. And you gave me an idea. The original prototype could fire the human safe round and the officer's the more powerful full bullet. Since to be an officer you just about have to be an aug. This gives me two versions, which I wanted to avoid, but it does make more sense. I have to admit you are the only one who has made me seriously consider just not giving it to my Character if they don't have augments.
        $endgroup$
        – neo
        May 24 at 15:18
















      $begingroup$
      Yes! That's exactly it. A rifle does exist and some nut made a handgun with it's cartridge. The government made a limited run of 100 or in my story. To give as decoration to officers. And you gave me an idea. The original prototype could fire the human safe round and the officer's the more powerful full bullet. Since to be an officer you just about have to be an aug. This gives me two versions, which I wanted to avoid, but it does make more sense. I have to admit you are the only one who has made me seriously consider just not giving it to my Character if they don't have augments.
      $endgroup$
      – neo
      May 24 at 15:18




      $begingroup$
      Yes! That's exactly it. A rifle does exist and some nut made a handgun with it's cartridge. The government made a limited run of 100 or in my story. To give as decoration to officers. And you gave me an idea. The original prototype could fire the human safe round and the officer's the more powerful full bullet. Since to be an officer you just about have to be an aug. This gives me two versions, which I wanted to avoid, but it does make more sense. I have to admit you are the only one who has made me seriously consider just not giving it to my Character if they don't have augments.
      $endgroup$
      – neo
      May 24 at 15:18











      2














      $begingroup$

      I can think of 3 ways to solve this problem:



      The first approach is to treat it more like a low calabre RPG and less like an actual bullet. Instead of an instantaneous combustion from a cartridge, making the rounds rocket propelled means that the shot accelerates over time significantly reducing the recoil at the moment of launch. 10lb anti-tank rounds can be fired such as to obtain nearly your desired speed within about 30 feet without creating bone breaking recoil. In contrast, a 19.25mm slug will be about 0.1-0.2lb so making a manageable projectile that can reach 800 ft/sec within inches of leaving your barrel should be pretty doable.



      A second approach is to treat it like heavy artillery by using hydraulic recoil compensation. By mounting the barrel and firing assembly to a hydraulic track, you can distribute the recoil over time as the barrel and firing assembly moves backwards on the track.



      A third approach is to make the slug an explosive round. Explosives are much less dense than lead; so, a 19.25mm explosive round would weight the same as a much smaller solid round; so, you would need a much smaller cartridge to get your shot up to speed.






      share|improve this answer











      $endgroup$















      • $begingroup$
        The 1500-3000 grain weight was actually the bullet weight not powder, Very helpful though on the HE aspect, I hadn't thought of that for this particular gun.
        $endgroup$
        – neo
        May 23 at 14:56










      • $begingroup$
        Both bullets and propellant are measured in grains. Since you was talking about downloading, I assumed he meant propellent, but in retrospect, those number are much too high to make since since for that.
        $endgroup$
        – Nosajimiki
        May 23 at 15:07












      • $begingroup$
        Your right, I should have been clearer even though they are too high for charge weights. Edited.
        $endgroup$
        – neo
        May 23 at 15:08
















      2














      $begingroup$

      I can think of 3 ways to solve this problem:



      The first approach is to treat it more like a low calabre RPG and less like an actual bullet. Instead of an instantaneous combustion from a cartridge, making the rounds rocket propelled means that the shot accelerates over time significantly reducing the recoil at the moment of launch. 10lb anti-tank rounds can be fired such as to obtain nearly your desired speed within about 30 feet without creating bone breaking recoil. In contrast, a 19.25mm slug will be about 0.1-0.2lb so making a manageable projectile that can reach 800 ft/sec within inches of leaving your barrel should be pretty doable.



      A second approach is to treat it like heavy artillery by using hydraulic recoil compensation. By mounting the barrel and firing assembly to a hydraulic track, you can distribute the recoil over time as the barrel and firing assembly moves backwards on the track.



      A third approach is to make the slug an explosive round. Explosives are much less dense than lead; so, a 19.25mm explosive round would weight the same as a much smaller solid round; so, you would need a much smaller cartridge to get your shot up to speed.






      share|improve this answer











      $endgroup$















      • $begingroup$
        The 1500-3000 grain weight was actually the bullet weight not powder, Very helpful though on the HE aspect, I hadn't thought of that for this particular gun.
        $endgroup$
        – neo
        May 23 at 14:56










      • $begingroup$
        Both bullets and propellant are measured in grains. Since you was talking about downloading, I assumed he meant propellent, but in retrospect, those number are much too high to make since since for that.
        $endgroup$
        – Nosajimiki
        May 23 at 15:07












      • $begingroup$
        Your right, I should have been clearer even though they are too high for charge weights. Edited.
        $endgroup$
        – neo
        May 23 at 15:08














      2














      2










      2







      $begingroup$

      I can think of 3 ways to solve this problem:



      The first approach is to treat it more like a low calabre RPG and less like an actual bullet. Instead of an instantaneous combustion from a cartridge, making the rounds rocket propelled means that the shot accelerates over time significantly reducing the recoil at the moment of launch. 10lb anti-tank rounds can be fired such as to obtain nearly your desired speed within about 30 feet without creating bone breaking recoil. In contrast, a 19.25mm slug will be about 0.1-0.2lb so making a manageable projectile that can reach 800 ft/sec within inches of leaving your barrel should be pretty doable.



      A second approach is to treat it like heavy artillery by using hydraulic recoil compensation. By mounting the barrel and firing assembly to a hydraulic track, you can distribute the recoil over time as the barrel and firing assembly moves backwards on the track.



      A third approach is to make the slug an explosive round. Explosives are much less dense than lead; so, a 19.25mm explosive round would weight the same as a much smaller solid round; so, you would need a much smaller cartridge to get your shot up to speed.






      share|improve this answer











      $endgroup$



      I can think of 3 ways to solve this problem:



      The first approach is to treat it more like a low calabre RPG and less like an actual bullet. Instead of an instantaneous combustion from a cartridge, making the rounds rocket propelled means that the shot accelerates over time significantly reducing the recoil at the moment of launch. 10lb anti-tank rounds can be fired such as to obtain nearly your desired speed within about 30 feet without creating bone breaking recoil. In contrast, a 19.25mm slug will be about 0.1-0.2lb so making a manageable projectile that can reach 800 ft/sec within inches of leaving your barrel should be pretty doable.



      A second approach is to treat it like heavy artillery by using hydraulic recoil compensation. By mounting the barrel and firing assembly to a hydraulic track, you can distribute the recoil over time as the barrel and firing assembly moves backwards on the track.



      A third approach is to make the slug an explosive round. Explosives are much less dense than lead; so, a 19.25mm explosive round would weight the same as a much smaller solid round; so, you would need a much smaller cartridge to get your shot up to speed.







      share|improve this answer














      share|improve this answer



      share|improve this answer








      edited May 23 at 15:08

























      answered May 23 at 14:51









      NosajimikiNosajimiki

      9,2691 gold badge14 silver badges46 bronze badges




      9,2691 gold badge14 silver badges46 bronze badges















      • $begingroup$
        The 1500-3000 grain weight was actually the bullet weight not powder, Very helpful though on the HE aspect, I hadn't thought of that for this particular gun.
        $endgroup$
        – neo
        May 23 at 14:56










      • $begingroup$
        Both bullets and propellant are measured in grains. Since you was talking about downloading, I assumed he meant propellent, but in retrospect, those number are much too high to make since since for that.
        $endgroup$
        – Nosajimiki
        May 23 at 15:07












      • $begingroup$
        Your right, I should have been clearer even though they are too high for charge weights. Edited.
        $endgroup$
        – neo
        May 23 at 15:08


















      • $begingroup$
        The 1500-3000 grain weight was actually the bullet weight not powder, Very helpful though on the HE aspect, I hadn't thought of that for this particular gun.
        $endgroup$
        – neo
        May 23 at 14:56










      • $begingroup$
        Both bullets and propellant are measured in grains. Since you was talking about downloading, I assumed he meant propellent, but in retrospect, those number are much too high to make since since for that.
        $endgroup$
        – Nosajimiki
        May 23 at 15:07












      • $begingroup$
        Your right, I should have been clearer even though they are too high for charge weights. Edited.
        $endgroup$
        – neo
        May 23 at 15:08
















      $begingroup$
      The 1500-3000 grain weight was actually the bullet weight not powder, Very helpful though on the HE aspect, I hadn't thought of that for this particular gun.
      $endgroup$
      – neo
      May 23 at 14:56




      $begingroup$
      The 1500-3000 grain weight was actually the bullet weight not powder, Very helpful though on the HE aspect, I hadn't thought of that for this particular gun.
      $endgroup$
      – neo
      May 23 at 14:56












      $begingroup$
      Both bullets and propellant are measured in grains. Since you was talking about downloading, I assumed he meant propellent, but in retrospect, those number are much too high to make since since for that.
      $endgroup$
      – Nosajimiki
      May 23 at 15:07






      $begingroup$
      Both bullets and propellant are measured in grains. Since you was talking about downloading, I assumed he meant propellent, but in retrospect, those number are much too high to make since since for that.
      $endgroup$
      – Nosajimiki
      May 23 at 15:07














      $begingroup$
      Your right, I should have been clearer even though they are too high for charge weights. Edited.
      $endgroup$
      – neo
      May 23 at 15:08




      $begingroup$
      Your right, I should have been clearer even though they are too high for charge weights. Edited.
      $endgroup$
      – neo
      May 23 at 15:08











      2














      $begingroup$

      Absolutely. The trick is, the bullet is a bomb, not a slug. The bullet is a miniature artillery shell with its own destructive mechanism - HEAT, bursting charge+shrapnel, etc.



      In fact, that's what requires the large bore - shrinking an "active, fuzed*" artillery shell down to 19mm is quite a challenge**, and this is why modern fighter jets shoot 25 or 30mm rounds.



      For Hellboy, this kind of destructive power makes sense given who he's fighting. It also makes it quite an achievement to make his own ammo - the miniaturization.



      So the bullet itself is doing most of the heavy lifting - it would be deadly if you handed it to the enemy (presuming this delivery method allowed the fuze* to operate normally).



      What does that do for kickback? The gun only needs to get the bullet to the destination. The destruction doesn't depend on imparting a whole bunch of kinetic energy like a normal bullet. That means the launching charge can be smaller, meaning lower kickback. However the large caliber shell necessitates a rather large gun with a lot of mass, which would soak up kickback, making it not bad at all. Anyone who could lift the gun could shoot it.



      When Hellboy says "it'll break your arm" he might be exaggerating, or might mean that he's also developed plain old kinetic-energy bullets with a lot of propellant. A bullet that wide would make no sense; surely it would be a sabot.





      * Do not edit. The mechanism which tells a bomb or shell when to detonate is called a fuZe. Look it up. A fuSe is something else.



      ** Part of what makes fuzes a challenge is making them not detonate accidentally. For instance the 16" rounds from Iowa arm from a) 1000++ gees from firing, b) -8 gees from air resistance, and spin from the rifling. Only then, impact+time delay sets it off.






      share|improve this answer











      $endgroup$




















        2














        $begingroup$

        Absolutely. The trick is, the bullet is a bomb, not a slug. The bullet is a miniature artillery shell with its own destructive mechanism - HEAT, bursting charge+shrapnel, etc.



        In fact, that's what requires the large bore - shrinking an "active, fuzed*" artillery shell down to 19mm is quite a challenge**, and this is why modern fighter jets shoot 25 or 30mm rounds.



        For Hellboy, this kind of destructive power makes sense given who he's fighting. It also makes it quite an achievement to make his own ammo - the miniaturization.



        So the bullet itself is doing most of the heavy lifting - it would be deadly if you handed it to the enemy (presuming this delivery method allowed the fuze* to operate normally).



        What does that do for kickback? The gun only needs to get the bullet to the destination. The destruction doesn't depend on imparting a whole bunch of kinetic energy like a normal bullet. That means the launching charge can be smaller, meaning lower kickback. However the large caliber shell necessitates a rather large gun with a lot of mass, which would soak up kickback, making it not bad at all. Anyone who could lift the gun could shoot it.



        When Hellboy says "it'll break your arm" he might be exaggerating, or might mean that he's also developed plain old kinetic-energy bullets with a lot of propellant. A bullet that wide would make no sense; surely it would be a sabot.





        * Do not edit. The mechanism which tells a bomb or shell when to detonate is called a fuZe. Look it up. A fuSe is something else.



        ** Part of what makes fuzes a challenge is making them not detonate accidentally. For instance the 16" rounds from Iowa arm from a) 1000++ gees from firing, b) -8 gees from air resistance, and spin from the rifling. Only then, impact+time delay sets it off.






        share|improve this answer











        $endgroup$


















          2














          2










          2







          $begingroup$

          Absolutely. The trick is, the bullet is a bomb, not a slug. The bullet is a miniature artillery shell with its own destructive mechanism - HEAT, bursting charge+shrapnel, etc.



          In fact, that's what requires the large bore - shrinking an "active, fuzed*" artillery shell down to 19mm is quite a challenge**, and this is why modern fighter jets shoot 25 or 30mm rounds.



          For Hellboy, this kind of destructive power makes sense given who he's fighting. It also makes it quite an achievement to make his own ammo - the miniaturization.



          So the bullet itself is doing most of the heavy lifting - it would be deadly if you handed it to the enemy (presuming this delivery method allowed the fuze* to operate normally).



          What does that do for kickback? The gun only needs to get the bullet to the destination. The destruction doesn't depend on imparting a whole bunch of kinetic energy like a normal bullet. That means the launching charge can be smaller, meaning lower kickback. However the large caliber shell necessitates a rather large gun with a lot of mass, which would soak up kickback, making it not bad at all. Anyone who could lift the gun could shoot it.



          When Hellboy says "it'll break your arm" he might be exaggerating, or might mean that he's also developed plain old kinetic-energy bullets with a lot of propellant. A bullet that wide would make no sense; surely it would be a sabot.





          * Do not edit. The mechanism which tells a bomb or shell when to detonate is called a fuZe. Look it up. A fuSe is something else.



          ** Part of what makes fuzes a challenge is making them not detonate accidentally. For instance the 16" rounds from Iowa arm from a) 1000++ gees from firing, b) -8 gees from air resistance, and spin from the rifling. Only then, impact+time delay sets it off.






          share|improve this answer











          $endgroup$



          Absolutely. The trick is, the bullet is a bomb, not a slug. The bullet is a miniature artillery shell with its own destructive mechanism - HEAT, bursting charge+shrapnel, etc.



          In fact, that's what requires the large bore - shrinking an "active, fuzed*" artillery shell down to 19mm is quite a challenge**, and this is why modern fighter jets shoot 25 or 30mm rounds.



          For Hellboy, this kind of destructive power makes sense given who he's fighting. It also makes it quite an achievement to make his own ammo - the miniaturization.



          So the bullet itself is doing most of the heavy lifting - it would be deadly if you handed it to the enemy (presuming this delivery method allowed the fuze* to operate normally).



          What does that do for kickback? The gun only needs to get the bullet to the destination. The destruction doesn't depend on imparting a whole bunch of kinetic energy like a normal bullet. That means the launching charge can be smaller, meaning lower kickback. However the large caliber shell necessitates a rather large gun with a lot of mass, which would soak up kickback, making it not bad at all. Anyone who could lift the gun could shoot it.



          When Hellboy says "it'll break your arm" he might be exaggerating, or might mean that he's also developed plain old kinetic-energy bullets with a lot of propellant. A bullet that wide would make no sense; surely it would be a sabot.





          * Do not edit. The mechanism which tells a bomb or shell when to detonate is called a fuZe. Look it up. A fuSe is something else.



          ** Part of what makes fuzes a challenge is making them not detonate accidentally. For instance the 16" rounds from Iowa arm from a) 1000++ gees from firing, b) -8 gees from air resistance, and spin from the rifling. Only then, impact+time delay sets it off.







          share|improve this answer














          share|improve this answer



          share|improve this answer








          edited May 26 at 17:44

























          answered May 26 at 15:44









          HarperHarper

          8,0511 gold badge12 silver badges28 bronze badges




          8,0511 gold badge12 silver badges28 bronze badges


























              1














              $begingroup$

              In fact, such guns were actually made, for instance Colt's 10-gauge revolving shotgun: https://www.forgottenweapons.com/colt-1855-10-gauge-revolving-shotgun-at-ria/



              (You may be confusing"revolver", which is a mechanism, with "pistol".)






              share|improve this answer









              $endgroup$




















                1














                $begingroup$

                In fact, such guns were actually made, for instance Colt's 10-gauge revolving shotgun: https://www.forgottenweapons.com/colt-1855-10-gauge-revolving-shotgun-at-ria/



                (You may be confusing"revolver", which is a mechanism, with "pistol".)






                share|improve this answer









                $endgroup$


















                  1














                  1










                  1







                  $begingroup$

                  In fact, such guns were actually made, for instance Colt's 10-gauge revolving shotgun: https://www.forgottenweapons.com/colt-1855-10-gauge-revolving-shotgun-at-ria/



                  (You may be confusing"revolver", which is a mechanism, with "pistol".)






                  share|improve this answer









                  $endgroup$



                  In fact, such guns were actually made, for instance Colt's 10-gauge revolving shotgun: https://www.forgottenweapons.com/colt-1855-10-gauge-revolving-shotgun-at-ria/



                  (You may be confusing"revolver", which is a mechanism, with "pistol".)







                  share|improve this answer












                  share|improve this answer



                  share|improve this answer










                  answered May 23 at 17:35









                  jamesqfjamesqf

                  11.3k1 gold badge19 silver badges39 bronze badges




                  11.3k1 gold badge19 silver badges39 bronze badges


































                      draft saved

                      draft discarded



















































                      Thanks for contributing an answer to Worldbuilding Stack Exchange!


                      • Please be sure to answer the question. Provide details and share your research!

                      But avoid



                      • Asking for help, clarification, or responding to other answers.

                      • Making statements based on opinion; back them up with references or personal experience.


                      Use MathJax to format equations. MathJax reference.


                      To learn more, see our tips on writing great answers.




                      draft saved


                      draft discarded














                      StackExchange.ready(
                      function () {
                      StackExchange.openid.initPostLogin('.new-post-login', 'https%3a%2f%2fworldbuilding.stackexchange.com%2fquestions%2f147616%2fcould-a-19-25mm-revolver-actually-exist%23new-answer', 'question_page');
                      }
                      );

                      Post as a guest















                      Required, but never shown





















































                      Required, but never shown














                      Required, but never shown












                      Required, but never shown







                      Required, but never shown

































                      Required, but never shown














                      Required, but never shown












                      Required, but never shown







                      Required, but never shown







                      Popular posts from this blog

                      He _____ here since 1970 . Answer needed [closed]What does “since he was so high” mean?Meaning of “catch birds for”?How do I ensure “since” takes the meaning I want?“Who cares here” meaningWhat does “right round toward” mean?the time tense (had now been detected)What does the phrase “ring around the roses” mean here?Correct usage of “visited upon”Meaning of “foiled rail sabotage bid”It was the third time I had gone to Rome or It is the third time I had been to Rome

                      Bunad

                      Færeyskur hestur Heimild | Tengill | Tilvísanir | LeiðsagnarvalRossið - síða um færeyska hrossið á færeyskuGott ár hjá færeyska hestinum